• Affiliate Program

Wordvice

  • UNITED STATES
  • 台灣 (TAIWAN)
  • TÜRKIYE (TURKEY)
  • Academic Editing Services
  • - Research Paper
  • - Journal Manuscript
  • - Dissertation
  • - College & University Assignments
  • Admissions Editing Services
  • - Application Essay
  • - Personal Statement
  • - Recommendation Letter
  • - Cover Letter
  • - CV/Resume
  • Business Editing Services
  • - Business Documents
  • - Report & Brochure
  • - Website & Blog
  • Writer Editing Services
  • - Script & Screenplay
  • Our Editors
  • Client Reviews
  • Editing & Proofreading Prices
  • Wordvice Points
  • Partner Discount
  • Plagiarism Checker
  • APA Citation Generator
  • MLA Citation Generator
  • Chicago Citation Generator
  • Vancouver Citation Generator
  • - APA Style
  • - MLA Style
  • - Chicago Style
  • - Vancouver Style
  • Writing & Editing Guide
  • Academic Resources
  • Admissions Resources

How to Write a Rhetorical Analysis Essay–Examples & Template

what is a rhetorical essay example

What is a Rhetorical Analysis Essay?

A rhetorical analysis essay is, as the name suggests, an analysis of someone else’s writing (or speech, or advert, or even cartoon) and how they use not only words but also rhetorical techniques to influence their audience in a certain way. A rhetorical analysis is less interested in what the author is saying and more in how they present it, what effect this has on their readers, whether they achieve their goals, and what approach they use to get there. 

Its structure is similar to that of most essays: An Introduction presents your thesis, a Body analyzes the text you have chosen, breaks it down into sections and explains how arguments have been constructed and how each part persuades, informs, or entertains the reader, and a Conclusion section sums up your evaluation. 

Note that your personal opinion on the matter is not relevant for your analysis and that you don’t state anywhere in your essay whether you agree or disagree with the stance the author takes.

In the following, we will define the key rhetorical concepts you need to write a good rhetorical analysis and give you some practical tips on where to start.

Key Rhetorical Concepts

Your goal when writing a rhetorical analysis is to think about and then carefully describe how the author has designed their text so that it has the intended effect on their audience. To do that, you need to consider a number of key rhetorical strategies: Rhetorical appeals (“Ethos”, “Logos”, and “Pathos”), context, as well as claims, supports, and warrants.

Ethos, Logos, and Pathos were introduced by Aristotle, way back in the 4th century BC, as the main ways in which language can be used to persuade an audience. They still represent the basis of any rhetorical analysis and are often referred to as the “rhetorical triangle”. 

These and other rhetorical techniques can all be combined to create the intended effect, and your job as the one analyzing a text is to break the writer’s arguments down and identify the concepts they are based on.

Rhetorical Appeals

Rhetorical appeal #1: ethos.

Ethos refers to the reputation or authority of the writer regarding the topic of their essay or speech and to how they use this to appeal to their audience. Just like we are more likely to buy a product from a brand or vendor we have confidence in than one we don’t know or have reason to distrust, Ethos-driven texts or speeches rely on the reputation of the author to persuade the reader or listener. When you analyze an essay, you should therefore look at how the writer establishes Ethos through rhetorical devices.

Does the author present themselves as an authority on their subject? If so, how? 

Do they highlight how impeccable their own behavior is to make a moral argument? 

Do they present themselves as an expert by listing their qualifications or experience to convince the reader of their opinion on something?

Rhetorical appeal #2: Pathos

The purpose of Pathos-driven rhetoric is to appeal to the reader’s emotions. A common example of pathos as a rhetorical means is adverts by charities that try to make you donate money to a “good cause”. To evoke the intended emotions in the reader, an author may use passionate language, tell personal stories, and employ vivid imagery so that the reader can imagine themselves in a certain situation and feel empathy with or anger towards others.

Rhetorical appeal #3: Logos

Logos, the “logical” appeal, uses reason to persuade. Reason and logic, supported by data, evidence, clearly defined methodology, and well-constructed arguments, are what most academic writing is based on. Emotions, those of the researcher/writer as well as those of the reader, should stay out of such academic texts, as should anyone’s reputation, beliefs, or personal opinions. 

Text and Context

To analyze a piece of writing, a speech, an advertisement, or even a satirical drawing, you need to look beyond the piece of communication and take the context in which it was created and/or published into account. 

Who is the person who wrote the text/drew the cartoon/designed the ad..? What audience are they trying to reach? Where was the piece published and what was happening there around that time? 

A political speech, for example, can be powerful even when read decades later, but the historical context surrounding it is an important aspect of the effect it was intended to have. 

Claims, Supports, and Warrants

To make any kind of argument, a writer needs to put forward specific claims, support them with data or evidence or even a moral or emotional appeal, and connect the dots logically so that the reader can follow along and agree with the points made.

The connections between statements, so-called “warrants”, follow logical reasoning but are not always clearly stated—the author simply assumes the reader understands the underlying logic, whether they present it “explicitly” or “implicitly”. Implicit warrants are commonly used in advertisements where seemingly happy people use certain products, wear certain clothes, accessories, or perfumes, or live certain lifestyles – with the connotation that, first, the product/perfume/lifestyle is what makes that person happy and, second, the reader wants to be as happy as the person in the ad. Some warrants are never clearly stated, and your job when writing a rhetorical analysis essay is therefore to identify them and bring them to light, to evaluate their validity, their effect on the reader, and the use of such means by the writer/creator. 

bust of plato the philosopher, rhetorical analysis essay

What are the Five Rhetorical Situations?

A “rhetorical situation” refers to the circumstance behind a text or other piece of communication that arises from a given context. It explains why a rhetorical piece was created, what its purpose is, and how it was constructed to achieve its aims.

Rhetorical situations can be classified into the following five categories:

Asking such questions when you analyze a text will help you identify all the aspects that play a role in the effect it has on its audience, and will allow you to evaluate whether it achieved its aims or where it may have failed to do so.

Rhetorical Analysis Essay Outline

Analyzing someone else’s work can seem like a big task, but as with every assignment or writing endeavor, you can break it down into smaller, well-defined steps that give you a practical structure to follow. 

To give you an example of how the different parts of your text may look when it’s finished, we will provide you with some excerpts from this rhetorical analysis essay example (which even includes helpful comments) published on the Online Writing Lab website of Excelsior University in Albany, NY. The text that this essay analyzes is this article on why one should or shouldn’t buy an Ipad. If you want more examples so that you can build your own rhetorical analysis template, have a look at this essay on Nabokov’s Lolita and the one provided here about the “Shitty First Drafts” chapter of Anne Lamott’s writing instruction book “Bird by Bird”.

Analyzing the Text

When writing a rhetorical analysis, you don’t choose the concepts or key points you think are relevant or want to address. Rather, you carefully read the text several times asking yourself questions like those listed in the last section on rhetorical situations to identify how the text “works” and how it was written to achieve that effect.

Start with focusing on the author : What do you think was their purpose for writing the text? Do they make one principal claim and then elaborate on that? Or do they discuss different topics? 

Then look at what audience they are talking to: Do they want to make a group of people take some action? Vote for someone? Donate money to a good cause? Who are these people? Is the text reaching this specific audience? Why or why not?

What tone is the author using to address their audience? Are they trying to evoke sympathy? Stir up anger? Are they writing from a personal perspective? Are they painting themselves as an authority on the topic? Are they using academic or informal language?

How does the author support their claims ? What kind of evidence are they presenting? Are they providing explicit or implicit warrants? Are these warrants valid or problematic? Is the provided evidence convincing?  

Asking yourself such questions will help you identify what rhetorical devices a text uses and how well they are put together to achieve a certain aim. Remember, your own opinion and whether you agree with the author are not the point of a rhetorical analysis essay – your task is simply to take the text apart and evaluate it.

If you are still confused about how to write a rhetorical analysis essay, just follow the steps outlined below to write the different parts of your rhetorical analysis: As every other essay, it consists of an Introduction , a Body (the actual analysis), and a Conclusion .

Rhetorical Analysis Introduction

The Introduction section briefly presents the topic of the essay you are analyzing, the author, their main claims, a short summary of the work by you, and your thesis statement . 

Tell the reader what the text you are going to analyze represents (e.g., historically) or why it is relevant (e.g., because it has become some kind of reference for how something is done). Describe what the author claims, asserts, or implies and what techniques they use to make their argument and persuade their audience. Finish off with your thesis statement that prepares the reader for what you are going to present in the next section – do you think that the author’s assumptions/claims/arguments were presented in a logical/appealing/powerful way and reached their audience as intended?

Have a look at an excerpt from the sample essay linked above to see what a rhetorical analysis introduction can look like. See how it introduces the author and article , the context in which it originally appeared , the main claims the author makes , and how this first paragraph ends in a clear thesis statement that the essay will then elaborate on in the following Body section:

Cory Doctorow ’s article on BoingBoing is an older review of the iPad , one of Apple’s most famous products. At the time of this article, however, the iPad was simply the latest Apple product to hit the market and was not yet so popular. Doctorow’s entire career has been entrenched in and around technology. He got his start as a CD-ROM programmer and is now a successful blogger and author. He is currently the co-editor of the BoingBoing blog on which this article was posted. One of his main points in this article comes from Doctorow’s passionate advocacy of free digital media sharing. He argues that the iPad is just another way for established technology companies to control our technological freedom and creativity . In “ Why I Won’t Buy an iPad (and Think You Shouldn’t, Either) ” published on Boing Boing in April of 2010, Cory Doctorow successfully uses his experience with technology, facts about the company Apple, and appeals to consumer needs to convince potential iPad buyers that Apple and its products, specifically the iPad, limit the digital rights of those who use them by controlling and mainstreaming the content that can be used and created on the device . 

Doing the Rhetorical Analysis

The main part of your analysis is the Body , where you dissect the text in detail. Explain what methods the author uses to inform, entertain, and/or persuade the audience. Use Aristotle’s rhetorical triangle and the other key concepts we introduced above. Use quotations from the essay to demonstrate what you mean. Work out why the writer used a certain approach and evaluate (and again, demonstrate using the text itself) how successful they were. Evaluate the effect of each rhetorical technique you identify on the audience and judge whether the effect is in line with the author’s intentions.

To make it easy for the reader to follow your thought process, divide this part of your essay into paragraphs that each focus on one strategy or one concept , and make sure they are all necessary and contribute to the development of your argument(s).

One paragraph of this section of your essay could, for example, look like this:

One example of Doctorow’s position is his comparison of Apple’s iStore to Wal-Mart. This is an appeal to the consumer’s logic—or an appeal to logos. Doctorow wants the reader to take his comparison and consider how an all-powerful corporation like the iStore will affect them. An iPad will only allow for apps and programs purchased through the iStore to be run on it; therefore, a customer must not only purchase an iPad but also any programs he or she wishes to use. Customers cannot create their own programs or modify the hardware in any way. 

As you can see, the author of this sample essay identifies and then explains to the reader how Doctorow uses the concept of Logos to appeal to his readers – not just by pointing out that he does it but by dissecting how it is done.

Rhetorical Analysis Conclusion

The conclusion section of your analysis should restate your main arguments and emphasize once more whether you think the author achieved their goal. Note that this is not the place to introduce new information—only rely on the points you have discussed in the body of your essay. End with a statement that sums up the impact the text has on its audience and maybe society as a whole:

Overall, Doctorow makes a good argument about why there are potentially many better things to drop a great deal of money on instead of the iPad. He gives some valuable information and facts that consumers should take into consideration before going out to purchase the new device. He clearly uses rhetorical tools to help make his case, and, overall, he is effective as a writer, even if, ultimately, he was ineffective in convincing the world not to buy an iPad . 

Frequently Asked Questions about Rhetorical Analysis Essays 

What is a rhetorical analysis essay.

A rhetorical analysis dissects a text or another piece of communication to work out and explain how it impacts its audience, how successfully it achieves its aims, and what rhetorical devices it uses to do that. 

While argumentative essays usually take a stance on a certain topic and argue for it, a rhetorical analysis identifies how someone else constructs their arguments and supports their claims.

What is the correct rhetorical analysis essay format?

Like most other essays, a rhetorical analysis contains an Introduction that presents the thesis statement, a Body that analyzes the piece of communication, explains how arguments have been constructed, and illustrates how each part persuades, informs, or entertains the reader, and a Conclusion section that summarizes the results of the analysis. 

What is the “rhetorical triangle”?

The rhetorical triangle was introduced by Aristotle as the main ways in which language can be used to persuade an audience: Logos appeals to the audience’s reason, Ethos to the writer’s status or authority, and Pathos to the reader’s emotions. Logos, Ethos, and Pathos can all be combined to create the intended effect, and your job as the one analyzing a text is to break the writer’s arguments down and identify what specific concepts each is based on.

Let Wordvice help you write a flawless rhetorical analysis essay! 

Whether you have to write a rhetorical analysis essay as an assignment or whether it is part of an application, our professional proofreading services feature professional editors are trained subject experts that make sure your text is in line with the required format, as well as help you improve the flow and expression of your writing. Let them be your second pair of eyes so that after receiving paper editing services or essay editing services from Wordvice, you can submit your manuscript or apply to the school of your dreams with confidence.

And check out our editing services for writers (including blog editing , script editing , and book editing ) to correct your important personal or business-related work.

  • Features for Creative Writers
  • Features for Work
  • Features for Higher Education
  • Features for Teachers
  • Features for Non-Native Speakers
  • Learn Blog Grammar Guide Community Events FAQ
  • Grammar Guide

What Is a Rhetorical Analysis and How to Write a Great One

Helly Douglas

Helly Douglas

Cover image for article

Do you have to write a rhetorical analysis essay? Fear not! We’re here to explain exactly what rhetorical analysis means, how you should structure your essay, and give you some essential “dos and don’ts.”

What is a Rhetorical Analysis Essay?

How do you write a rhetorical analysis, what are the three rhetorical strategies, what are the five rhetorical situations, how to plan a rhetorical analysis essay, creating a rhetorical analysis essay, examples of great rhetorical analysis essays, final thoughts.

A rhetorical analysis essay studies how writers and speakers have used words to influence their audience. Think less about the words the author has used and more about the techniques they employ, their goals, and the effect this has on the audience.

Image showing definitions

In your analysis essay, you break a piece of text (including cartoons, adverts, and speeches) into sections and explain how each part works to persuade, inform, or entertain. You’ll explore the effectiveness of the techniques used, how the argument has been constructed, and give examples from the text.

A strong rhetorical analysis evaluates a text rather than just describes the techniques used. You don’t include whether you personally agree or disagree with the argument.

Structure a rhetorical analysis in the same way as most other types of academic essays . You’ll have an introduction to present your thesis, a main body where you analyze the text, which then leads to a conclusion.

Think about how the writer (also known as a rhetor) considers the situation that frames their communication:

  • Topic: the overall purpose of the rhetoric
  • Audience: this includes primary, secondary, and tertiary audiences
  • Purpose: there are often more than one to consider
  • Context and culture: the wider situation within which the rhetoric is placed

Back in the 4th century BC, Aristotle was talking about how language can be used as a means of persuasion. He described three principal forms —Ethos, Logos, and Pathos—often referred to as the Rhetorical Triangle . These persuasive techniques are still used today.

Image showing rhetorical strategies

Rhetorical Strategy 1: Ethos

Are you more likely to buy a car from an established company that’s been an important part of your community for 50 years, or someone new who just started their business?

Reputation matters. Ethos explores how the character, disposition, and fundamental values of the author create appeal, along with their expertise and knowledge in the subject area.

Aristotle breaks ethos down into three further categories:

  • Phronesis: skills and practical wisdom
  • Arete: virtue
  • Eunoia: goodwill towards the audience

Ethos-driven speeches and text rely on the reputation of the author. In your analysis, you can look at how the writer establishes ethos through both direct and indirect means.

Rhetorical Strategy 2: Pathos

Pathos-driven rhetoric hooks into our emotions. You’ll often see it used in advertisements, particularly by charities wanting you to donate money towards an appeal.

Common use of pathos includes:

  • Vivid description so the reader can imagine themselves in the situation
  • Personal stories to create feelings of empathy
  • Emotional vocabulary that evokes a response

By using pathos to make the audience feel a particular emotion, the author can persuade them that the argument they’re making is compelling.

Rhetorical Strategy 3: Logos

Logos uses logic or reason. It’s commonly used in academic writing when arguments are created using evidence and reasoning rather than an emotional response. It’s constructed in a step-by-step approach that builds methodically to create a powerful effect upon the reader.

Rhetoric can use any one of these three techniques, but effective arguments often appeal to all three elements.

The rhetorical situation explains the circumstances behind and around a piece of rhetoric. It helps you think about why a text exists, its purpose, and how it’s carried out.

Image showing 5 rhetorical situations

The rhetorical situations are:

  • 1) Purpose: Why is this being written? (It could be trying to inform, persuade, instruct, or entertain.)
  • 2) Audience: Which groups or individuals will read and take action (or have done so in the past)?
  • 3) Genre: What type of writing is this?
  • 4) Stance: What is the tone of the text? What position are they taking?
  • 5) Media/Visuals: What means of communication are used?

Understanding and analyzing the rhetorical situation is essential for building a strong essay. Also think about any rhetoric restraints on the text, such as beliefs, attitudes, and traditions that could affect the author's decisions.

Before leaping into your essay, it’s worth taking time to explore the text at a deeper level and considering the rhetorical situations we looked at before. Throw away your assumptions and use these simple questions to help you unpick how and why the text is having an effect on the audience.

Image showing what to consider when planning a rhetorical essay

1: What is the Rhetorical Situation?

  • Why is there a need or opportunity for persuasion?
  • How do words and references help you identify the time and location?
  • What are the rhetoric restraints?
  • What historical occasions would lead to this text being created?

2: Who is the Author?

  • How do they position themselves as an expert worth listening to?
  • What is their ethos?
  • Do they have a reputation that gives them authority?
  • What is their intention?
  • What values or customs do they have?

3: Who is it Written For?

  • Who is the intended audience?
  • How is this appealing to this particular audience?
  • Who are the possible secondary and tertiary audiences?

4: What is the Central Idea?

  • Can you summarize the key point of this rhetoric?
  • What arguments are used?
  • How has it developed a line of reasoning?

5: How is it Structured?

  • What structure is used?
  • How is the content arranged within the structure?

6: What Form is Used?

  • Does this follow a specific literary genre?
  • What type of style and tone is used, and why is this?
  • Does the form used complement the content?
  • What effect could this form have on the audience?

7: Is the Rhetoric Effective?

  • Does the content fulfil the author’s intentions?
  • Does the message effectively fit the audience, location, and time period?

Once you’ve fully explored the text, you’ll have a better understanding of the impact it’s having on the audience and feel more confident about writing your essay outline.

A great essay starts with an interesting topic. Choose carefully so you’re personally invested in the subject and familiar with it rather than just following trending topics. There are lots of great ideas on this blog post by My Perfect Words if you need some inspiration. Take some time to do background research to ensure your topic offers good analysis opportunities.

Image showing considerations for a rhetorical analysis topic

Remember to check the information given to you by your professor so you follow their preferred style guidelines. This outline example gives you a general idea of a format to follow, but there will likely be specific requests about layout and content in your course handbook. It’s always worth asking your institution if you’re unsure.

Make notes for each section of your essay before you write. This makes it easy for you to write a well-structured text that flows naturally to a conclusion. You will develop each note into a paragraph. Look at this example by College Essay for useful ideas about the structure.

Image showing how to structure an essay

1: Introduction

This is a short, informative section that shows you understand the purpose of the text. It tempts the reader to find out more by mentioning what will come in the main body of your essay.

  • Name the author of the text and the title of their work followed by the date in parentheses
  • Use a verb to describe what the author does, e.g. “implies,” “asserts,” or “claims”
  • Briefly summarize the text in your own words
  • Mention the persuasive techniques used by the rhetor and its effect

Create a thesis statement to come at the end of your introduction.

After your introduction, move on to your critical analysis. This is the principal part of your essay.

  • Explain the methods used by the author to inform, entertain, and/or persuade the audience using Aristotle's rhetorical triangle
  • Use quotations to prove the statements you make
  • Explain why the writer used this approach and how successful it is
  • Consider how it makes the audience feel and react

Make each strategy a new paragraph rather than cramming them together, and always use proper citations. Check back to your course handbook if you’re unsure which citation style is preferred.

3: Conclusion

Your conclusion should summarize the points you’ve made in the main body of your essay. While you will draw the points together, this is not the place to introduce new information you’ve not previously mentioned.

Use your last sentence to share a powerful concluding statement that talks about the impact the text has on the audience(s) and wider society. How have its strategies helped to shape history?

Before You Submit

Poor spelling and grammatical errors ruin a great essay. Use ProWritingAid to check through your finished essay before you submit. It will pick up all the minor errors you’ve missed and help you give your essay a final polish. Look at this useful ProWritingAid webinar for further ideas to help you significantly improve your essays. Sign up for a free trial today and start editing your essays!

Screenshot of ProWritingAid's web editor

You’ll find countless examples of rhetorical analysis online, but they range widely in quality. Your institution may have example essays they can share with you to show you exactly what they’re looking for.

The following links should give you a good starting point if you’re looking for ideas:

Pearson Canada has a range of good examples. Look at how embedded quotations are used to prove the points being made. The end questions help you unpick how successful each essay is.

Excelsior College has an excellent sample essay complete with useful comments highlighting the techniques used.

Brighton Online has a selection of interesting essays to look at. In this specific example, consider how wider reading has deepened the exploration of the text.

Image showing tips when reading a sample essay

Writing a rhetorical analysis essay can seem daunting, but spending significant time deeply analyzing the text before you write will make it far more achievable and result in a better-quality essay overall.

It can take some time to write a good essay. Aim to complete it well before the deadline so you don’t feel rushed. Use ProWritingAid’s comprehensive checks to find any errors and make changes to improve readability. Then you’ll be ready to submit your finished essay, knowing it’s as good as you can possibly make it.

Try ProWritingAid's Editor for Yourself

what is a rhetorical essay example

Be confident about grammar

Check every email, essay, or story for grammar mistakes. Fix them before you press send.

Helly Douglas is a UK writer and teacher, specialising in education, children, and parenting. She loves making the complex seem simple through blogs, articles, and curriculum content. You can check out her work at hellydouglas.com or connect on Twitter @hellydouglas. When she’s not writing, you will find her in a classroom, being a mum or battling against the wilderness of her garden—the garden is winning!

Get started with ProWritingAid

Drop us a line or let's stay in touch via :

Reference management. Clean and simple.

How to write a rhetorical analysis

Rhetorical analysis illustration

What is a rhetorical analysis?

What are the key concepts of a rhetorical analysis, rhetorical situation, claims, supports, and warrants.

  • Step 1: Plan and prepare
  • Step 2: Write your introduction
  • Step 3: Write the body
  • Step 4: Write your conclusion

Frequently Asked Questions about rhetorical analysis

Related articles.

Rhetoric is the art of persuasion and aims to study writers’ or speakers' techniques to inform, persuade, or motivate their audience. Thus, a rhetorical analysis aims to explore the goals and motivations of an author, the techniques they’ve used to reach their audience, and how successful these techniques were.

This will generally involve analyzing a specific text and considering the following aspects to connect the rhetorical situation to the text:

  • Does the author successfully support the thesis or claims made in the text? Here, you’ll analyze whether the author holds to their argument consistently throughout the text or whether they wander off-topic at some point.
  • Does the author use evidence effectively considering the text’s intended audience? Here, you’ll consider the evidence used by the author to support their claims and whether the evidence resonates with the intended audience.
  • What rhetorical strategies the author uses to achieve their goals. Here, you’ll consider the word choices by the author and whether these word choices align with their agenda for the text.
  • The tone of the piece. Here, you’ll consider the tone used by the author in writing the piece by looking at specific words and aspects that set the tone.
  • Whether the author is objective or trying to convince the audience of a particular viewpoint. When it comes to objectivity, you’ll consider whether the author is objective or holds a particular viewpoint they want to convince the audience of. If they are, you’ll also consider whether their persuasion interferes with how the text is read and understood.
  • Does the author correctly identify the intended audience? It’s important to consider whether the author correctly writes the text for the intended audience and what assumptions the author makes about the audience.
  • Does the text make sense? Here, you’ll consider whether the author effectively reasons, based on the evidence, to arrive at the text’s conclusion.
  • Does the author try to appeal to the audience’s emotions? You’ll need to consider whether the author uses any words, ideas, or techniques to appeal to the audience’s emotions.
  • Can the author be believed? Finally, you’ll consider whether the audience will accept the arguments and ideas of the author and why.

Summing up, unlike summaries that focus on what an author said, a rhetorical analysis focuses on how it’s said, and it doesn’t rely on an analysis of whether the author was right or wrong but rather how they made their case to arrive at their conclusions.

Although rhetorical analysis is most used by academics as part of scholarly work, it can be used to analyze any text including speeches, novels, television shows or films, advertisements, or cartoons.

Now that we’ve seen what rhetorical analysis is, let’s consider some of its key concepts .

Any rhetorical analysis starts with the rhetorical situation which identifies the relationships between the different elements of the text. These elements include the audience, author or writer, the author’s purpose, the delivery method or medium, and the content:

  • Audience: The audience is simply the readers of a specific piece of text or content or printed material. For speeches or other mediums like film and video, the audience would be the listeners or viewers. Depending on the specific piece of text or the author’s perception, the audience might be real, imagined, or invoked. With a real audience, the author writes to the people actually reading or listening to the content while, for an imaginary audience, the author writes to an audience they imagine would read the content. Similarly, for an invoked audience, the author writes explicitly to a specific audience.
  • Author or writer: The author or writer, also commonly referred to as the rhetor in the context of rhetorical analysis, is the person or the group of persons who authored the text or content.
  • The author’s purpose: The author’s purpose is the author’s reason for communicating to the audience. In other words, the author’s purpose encompasses what the author expects or intends to achieve with the text or content.
  • Alphabetic text includes essays, editorials, articles, speeches, and other written pieces.
  • Imaging includes website and magazine advertisements, TV commercials, and the like.
  • Audio includes speeches, website advertisements, radio or tv commercials, or podcasts.
  • Context: The context of the text or content considers the time, place, and circumstances surrounding the delivery of the text to its audience. With respect to context, it might often also be helpful to analyze the text in a different context to determine its impact on a different audience and in different circumstances.

An author will use claims, supports, and warrants to build the case around their argument, irrespective of whether the argument is logical and clearly defined or needs to be inferred by the audience:

  • Claim: The claim is the main idea or opinion of an argument that the author must prove to the intended audience. In other words, the claim is the fact or facts the author wants to convince the audience of. Claims are usually explicitly stated but can, depending on the specific piece of content or text, be implied from the content. Although these claims could be anything and an argument may be based on a single or several claims, the key is that these claims should be debatable.
  • Support: The supports are used by the author to back up the claims they make in their argument. These supports can include anything from fact-based, objective evidence to subjective emotional appeals and personal experiences used by the author to convince the audience of a specific claim. Either way, the stronger and more reliable the supports, the more likely the audience will be to accept the claim.
  • Warrant: The warrants are the logic and assumptions that connect the supports to the claims. In other words, they’re the assumptions that make the initial claim possible. The warrant is often unstated, and the author assumes that the audience will be able to understand the connection between the claims and supports. In turn, this is based on the author’s assumption that they share a set of values and beliefs with the audience that will make them understand the connection mentioned above. Conversely, if the audience doesn’t share these beliefs and values with the author, the argument will not be that effective.

Appeals are used by authors to convince their audience and, as such, are an integral part of the rhetoric and are often referred to as the rhetorical triangle. As a result, an author may combine all three appeals to convince their audience:

  • Ethos: Ethos represents the authority or credibility of the author. To be successful, the author needs to convince the audience of their authority or credibility through the language and delivery techniques they use. This will, for example, be the case where an author writing on a technical subject positions themselves as an expert or authority by referring to their qualifications or experience.
  • Logos: Logos refers to the reasoned argument the author uses to persuade their audience. In other words, it refers to the reasons or evidence the author proffers in substantiation of their claims and can include facts, statistics, and other forms of evidence. For this reason, logos is also the dominant approach in academic writing where authors present and build up arguments using reasoning and evidence.
  • Pathos: Through pathos, also referred to as the pathetic appeal, the author attempts to evoke the audience’s emotions through the use of, for instance, passionate language, vivid imagery, anger, sympathy, or any other emotional response.

To write a rhetorical analysis, you need to follow the steps below:

With a rhetorical analysis, you don’t choose concepts in advance and apply them to a specific text or piece of content. Rather, you’ll have to analyze the text to identify the separate components and plan and prepare your analysis accordingly.

Here, it might be helpful to use the SOAPSTone technique to identify the components of the work. SOAPSTone is a common acronym in analysis and represents the:

  • Speaker . Here, you’ll identify the author or the narrator delivering the content to the audience.
  • Occasion . With the occasion, you’ll identify when and where the story takes place and what the surrounding context is.
  • Audience . Here, you’ll identify who the audience or intended audience is.
  • Purpose . With the purpose, you’ll need to identify the reason behind the text or what the author wants to achieve with their writing.
  • Subject . You’ll also need to identify the subject matter or topic of the text.
  • Tone . The tone identifies the author’s feelings towards the subject matter or topic.

Apart from gathering the information and analyzing the components mentioned above, you’ll also need to examine the appeals the author uses in writing the text and attempting to persuade the audience of their argument. Moreover, you’ll need to identify elements like word choice, word order, repetition, analogies, and imagery the writer uses to get a reaction from the audience.

Once you’ve gathered the information and examined the appeals and strategies used by the author as mentioned above, you’ll need to answer some questions relating to the information you’ve collected from the text. The answers to these questions will help you determine the reasons for the choices the author made and how well these choices support the overall argument.

Here, some of the questions you’ll ask include:

  • What was the author’s intention?
  • Who was the intended audience?
  • What is the author’s argument?
  • What strategies does the author use to build their argument and why do they use those strategies?
  • What appeals the author uses to convince and persuade the audience?
  • What effect the text has on the audience?

Keep in mind that these are just some of the questions you’ll ask, and depending on the specific text, there might be others.

Once you’ve done your preparation, you can start writing the rhetorical analysis. It will start off with an introduction which is a clear and concise paragraph that shows you understand the purpose of the text and gives more information about the author and the relevance of the text.

The introduction also summarizes the text and the main ideas you’ll discuss in your analysis. Most importantly, however, is your thesis statement . This statement should be one sentence at the end of the introduction that summarizes your argument and tempts your audience to read on and find out more about it.

After your introduction, you can proceed with the body of your analysis. Here, you’ll write at least three paragraphs that explain the strategies and techniques used by the author to convince and persuade the audience, the reasons why the writer used this approach, and why it’s either successful or unsuccessful.

You can structure the body of your analysis in several ways. For example, you can deal with every strategy the author uses in a new paragraph, but you can also structure the body around the specific appeals the author used or chronologically.

No matter how you structure the body and your paragraphs, it’s important to remember that you support each one of your arguments with facts, data, examples, or quotes and that, at the end of every paragraph, you tie the topic back to your original thesis.

Finally, you’ll write the conclusion of your rhetorical analysis. Here, you’ll repeat your thesis statement and summarize the points you’ve made in the body of your analysis. Ultimately, the goal of the conclusion is to pull the points of your analysis together so you should be careful to not raise any new issues in your conclusion.

After you’ve finished your conclusion, you’ll end your analysis with a powerful concluding statement of why your argument matters and an invitation to conduct more research if needed.

A rhetorical analysis aims to explore the goals and motivations of an author, the techniques they’ve used to reach their audience, and how successful these techniques were. Although rhetorical analysis is most used by academics as part of scholarly work, it can be used to analyze any text including speeches, novels, television shows or films, advertisements, or cartoons.

The steps to write a rhetorical analysis include:

Your rhetorical analysis introduction is a clear and concise paragraph that shows you understand the purpose of the text and gives more information about the author and the relevance of the text. The introduction also summarizes the text and the main ideas you’ll discuss in your analysis.

Ethos represents the authority or credibility of the author. To be successful, the author needs to convince the audience of their authority or credibility through the language and delivery techniques they use. This will, for example, be the case where an author writing on a technical subject positions themselves as an expert or authority by referring to their qualifications or experience.

Appeals are used by authors to convince their audience and, as such, are an integral part of the rhetoric and are often referred to as the rhetorical triangle. The 3 types of appeals are ethos, logos, and pathos.

what is a rhetorical essay example

  • Link to facebook
  • Link to linkedin
  • Link to twitter
  • Link to youtube
  • Writing Tips

How to Write a Rhetorical Analysis Essay

How to Write a Rhetorical Analysis Essay

3-minute read

  • 22nd August 2023

A rhetorical analysis essay is a type of academic writing that analyzes how authors use language, persuasion techniques , and other rhetorical strategies to communicate with their audience. In this post, we’ll review how to write a rhetorical analysis essay, including:

  • Understanding the assignment guidelines
  • Introducing your essay topic
  • Examining the rhetorical strategies
  • Summarizing your main points

Keep reading for a step-by-step guide to rhetorical analysis.

What Is a Rhetorical Strategy?

A rhetorical strategy is a deliberate approach or technique a writer uses to convey a message and/or persuade the audience. A rhetorical strategy typically involves using language, sentence structure, and tone/style to influence the audience to think a certain way or understand a specific point of view. Rhetorical strategies are especially common in advertisements, speeches, and political writing, but you can also find them in many other types of literature.

1.   Understanding the Assignment Guidelines

Before you begin your rhetorical analysis essay, make sure you understand the assignment and guidelines. Typically, when writing a rhetorical analysis, you should approach the text objectively, focusing on the techniques the author uses rather than expressing your own opinions about the topic or summarizing the content. Thus, it’s essential to discuss the rhetorical methods used and then back up your analysis with evidence and quotations from the text.

2.   Introducing Your Essay Topic

Introduce your essay by providing some context about the text you’re analyzing. Give a brief overview of the author, intended audience, and purpose of the writing. You should also clearly state your thesis , which is your main point or argument about how and why the author uses rhetorical strategies. Try to avoid going into detail on any points or diving into specific examples – the introduction should be concise, and you’ll be providing a much more in-depth analysis later in the text.

3.   Examining the Rhetorical Strategies

In the body paragraphs, analyze the rhetorical strategies the author uses. Here are some common rhetorical strategies to include in your discussion:

●  Ethos : Establishing trust between the writer and the audience by appealing to credibility and ethics

●  Pathos : Appealing to the audience’s emotions and values

●  Logos : Employing logic, reason, and evidence to appeal to the reader

Find this useful?

Subscribe to our newsletter and get writing tips from our editors straight to your inbox.

●  Diction : Deliberately choosing specific language and vocabulary

●  Syntax : Structuring and arranging sentences in certain ways

●  Tone : Conveying attitude or mood in certain ways

●  Literary Devices : Using metaphors, similes, analogies , repetition, etc.

Keep in mind that for a rhetorical analysis essay, you’re not usually required to find examples of all of the above rhetorical strategies. But for each one you do analyze, consider how it contributes to the author’s purpose, how it influences the audience, and what emotions or thoughts it could evoke in the reader.

4.   Summarizing Your Main Points

In your conclusion , sum up the main points of your analysis and restate your thesis. Without introducing any new points (such as topics or ideas you haven’t already covered in the main body of your essay), summarize the overall impact that the author’s rhetorical strategies likely had on their intended audience.

Expert Proofreading Services

Ensure that your rhetorical analysis essay stands out by having our expert team proofread it. Send in your free sample of 500 words or less and see for yourself the difference in your work!

Share this article:

Post A New Comment

Got content that needs a quick turnaround? Let us polish your work. Explore our editorial business services.

How to insert a text box in a google doc.

Google Docs is a powerful collaborative tool, and mastering its features can significantly enhance your...

2-minute read

How to Cite the CDC in APA

If you’re writing about health issues, you might need to reference the Centers for Disease...

5-minute read

Six Product Description Generator Tools for Your Product Copy

Introduction If you’re involved with ecommerce, you’re likely familiar with the often painstaking process of...

What Is a Content Editor?

Are you interested in learning more about the role of a content editor and the...

4-minute read

The Benefits of Using an Online Proofreading Service

Proofreading is important to ensure your writing is clear and concise for your readers. Whether...

6 Online AI Presentation Maker Tools

Creating presentations can be time-consuming and frustrating. Trying to construct a visually appealing and informative...

Logo Harvard University

Make sure your writing is the best it can be with our expert English proofreading and editing.

what is a rhetorical essay example

How to Write a Rhetorical Analysis: Full Guide

what is a rhetorical essay example

Have you ever been completely fascinated by a speech or ad, wondering how it managed to convince you so effectively? From powerful political speeches to catchy commercials, persuasion is all around us, shaping our thoughts and choices every day.

In this guide, we'll explain all about a rhetorical analysis essay. We'll break down the clever tricks writers and speakers use to win over their audience, like how they choose their words carefully and play with our emotions. This article will give you the tools you need to understand and analyze texts more deeply. So, let’s jump right in and start by understanding the nature of this assignment first.

What is a Rhetorical Analysis Essay

A rhetorical analysis essay is a type of essay where you examine how authors or speakers use words, phrases, and other techniques to influence or persuade their audience. This type of essay focuses on analyzing the strategies used by the writer or speaker to achieve their purpose, whether it's to inform, persuade, entertain, or provoke.

You'll dissect the text or speech into its components, looking at how each part contributes to the overall message. This might involve examining the introduction, thesis statement, body paragraphs, evidence, and conclusion.

Once you've identified the strategies used, you'll assess their effectiveness in achieving the author's or speaker's purpose. This involves considering the intended audience, context, and the impact of the communication.

As per our essay writing service , some common topics for rhetorical analysis include analyzing speeches by influential leaders, dissecting political advertisements, or examining the rhetoric used in literary works.

Do You Want to Ease Your Academic Burden?

Order a rhetorical analysis essay from our expert writers today and experience the power of top-notch academic writing.

Rhetorical Analysis Topic Ideas

Now that we've grasped the essence of a rhetorical analysis essay let's explore some potential topics you might consider for your own analysis. Here are 15 specific ideas to get you started:

  • The Use of Metaphors in Barack Obama's 'Yes We Can' Speech
  • Visual Rhetoric in Dove's 'Real Beauty' Advertising Campaign
  • The Role of Irony in Jonathan Swift's 'A Modest Proposal'
  • The Manipulation of Emotions in Coca-Cola's 'Share a Coke' Campaign
  • The Repetition Technique in Winston Churchill's 'We Shall Fight on the Beaches' Speech
  • The Argument Structure in Michelle Obama's Speech on Education
  • The Use of Imagery in Edgar Allan Poe's 'The Raven'
  • Gender Stereotypes in Old Spice's 'The Man Your Man Could Smell Like' Ad
  • Satirical Elements in George Orwell's 'Animal Farm'
  • The Influence of Tone in Greta Thunberg's Climate Change Speeches
  • Political Symbolism in Banksy's Street Art
  • Humor as Persuasion in Ellen DeGeneres' Stand-Up Comedy
  • The Power of Silence in Emma Watson's UN Speech on Gender Equality
  • Ethical Appeals in ASPCA's Animal Rights Advertisements
  • The Cultural References in Super Bowl Commercials: A Case Study

How to Write a Rhetorical Analysis

Understanding how to start a rhetorical analysis essay involves dissecting a piece of communication to learn how it works and what effect it aims to achieve. This analytical process typically includes five paragraphs and three main parts: an introduction, three body paragraphs, and a conclusion. Below, our analytical essay writing service will explain each in more detail

Rhetorical Analysis Essay Outline

Major Rhetorical Elements

Before heading towards the analysis process, it's essential to grasp some key rhetorical concepts that will help guide your examination of the text or speech. These concepts provide a framework for understanding how authors and speakers use language to persuade and influence their audience.

Ethos, pathos logos in rhetorical analysis form the foundation of persuasive communication and are often intertwined in rhetorical strategies. Ethos refers to the credibility or authority of the speaker or author. Pathos involves appealing to the audience's emotions, while logos appeals to reason and logic.

There are also other rhetorical devices that are specific techniques or patterns of language used to convey meaning or evoke particular responses. Examples include metaphor, simile, imagery, irony, repetition, and hyperbole. Recognizing and analyzing these devices can provide insight into the author's intended message and its impact on the audience.

Tone and mood also play crucial roles in shaping the audience's perception and response to the communication. Tone refers to the author's attitude toward the subject matter, while mood describes the emotional atmosphere created by the text.

Whether you ask us - write my essay , or tackle the task yourself, familiarizing yourself with these concepts will help you analyze the text and persuade the audience more effectively.

Understanding Rhetorical Appeals

Understanding Rhetorical Appeals

First off, what is ethos in rhetorical analysis? Well, it revolves around establishing the credibility and authority of the speaker or author. This appeal seeks to convince the audience that the communicator is trustworthy, knowledgeable, and reliable. Ethos in rhetorical analysis can be built through various means, including:

  • Professional Credentials : Demonstrating expertise in the subject matter through relevant qualifications or experience.
  • Personal Character : Highlighting traits such as honesty, integrity, and sincerity to engender trust and respect.
  • Association : Aligning oneself with respected individuals, institutions, or causes to enhance credibility by association.

For instance, in a health-related speech, a doctor might leverage their medical expertise and professional experience (credentials) to establish ethos. Similarly, a celebrity endorsing a product is using their fame and reputation (association) to persuade consumers.

Now, let's understand what is pathos in rhetorical analysis. Pathos involves appealing to the audience's emotions, aiming to evoke feelings such as empathy, sympathy, joy, anger, or fear. This emotional connection can be a powerful tool for persuasion, as it resonates with the audience on a personal level. Strategies for employing pathos in rhetorical analysis include:

  • Vivid Imagery : Painting a vivid picture or narrative that elicits strong emotional responses from the audience.
  • Anecdotes : Sharing personal stories or anecdotes that evoke empathy or sympathy and make the message more relatable.
  • Language Choice : Using emotive language, sensory details, and rhetorical devices to evoke specific emotional reactions.

For example, in a charity advertisement for children in need, images of impoverished and suffering children coupled with heart-wrenching stories (anecdotes) are used to evoke feelings of compassion and a desire to help.

Lastly, what is logos in rhetorical analysis, you may ask. It appeals to reason and logic, aiming to persuade the audience through rational argumentation and evidence. This appeal relies on facts, statistics, logical reasoning, and sound arguments to convince the audience of the validity of the message. Strategies for employing logos in rhetorical analysis include:

  • Factual Evidence : Providing empirical data, research findings, or expert opinions to support the argument.
  • Logical Reasoning : Presenting a well-structured argument with clear premises and conclusions that logically follow one another.
  • Counterarguments : Addressing potential counterarguments and refuting them with logical reasoning and evidence.

For instance, in a persuasive essay advocating for environmental conservation, the author might present scientific data on climate change (factual evidence) and use logical reasoning to explain the consequences of inaction.

Text and Context

Text analysis involves closely examining the language, structure, and rhetorical devices employed within the communication. This includes identifying key themes, rhetorical appeals, persuasive strategies, and stylistic elements used by the author or speaker to convey their message.

For example, in a political speech advocating for healthcare reform, text analysis might involve identifying the use of rhetorical appeals such as ethos (e.g., highlighting the speaker's experience in healthcare policy), pathos (e.g., sharing anecdotes of individuals struggling with medical costs), and logos (e.g., presenting statistics on healthcare affordability).

Contextual analysis involves considering the broader social, cultural, and historical factors that shape communication and influence its reception. This includes examining the audience demographics, the political and cultural climate, the historical events surrounding the communication, and any relevant societal norms or values.

For instance, when analyzing a historical speech advocating for civil rights, contextual research paper writers might involve considering the social and political context of the time, including prevailing attitudes towards race, ongoing civil rights movements, and recent legislative developments.

Claims, Supports, and Warrants

A claim is a statement or assertion that the author or speaker is advocating for or seeking to prove. Claims can take various forms, including factual claims (assertions of fact), value claims (judgments about what is good or bad), and policy claims (proposals for action). For example, in an argumentative essay about the importance of exercise, the claim might be that regular physical activity is essential for maintaining good health.

Supports are the evidence, reasoning, or examples provided to substantiate and strengthen the claims being made. Supports can take many forms, including empirical data, expert testimony, personal anecdotes, logical reasoning, and analogies. The quality and relevance of the supports provided play a critical role in the persuasiveness of the argument.

Continuing with the example of the argumentative essay about exercise, supports might include scientific studies demonstrating the health benefits of physical activity, testimonials from fitness experts, and personal stories of individuals who have experienced positive changes from incorporating exercise into their routine.

Warrants are the underlying assumptions or principles that connect the supports to the claims. They provide the reasoning or justification for why the supports are relevant and valid evidence for supporting the claims. Warrants are often implicit rather than explicit and require careful analysis to uncover. In the context of the essay on exercise, the warrant connecting the supports to the claim might be the assumption that actions that promote good health are inherently valuable and worthy of pursuit.

Rhetorical Analysis Essay Outline

Whether you opt for the option to buy essay or start writing it yourself, it's important to use a clear plan to organize your thoughts well. This plan usually includes four main steps, each looking at different parts of your analysis.

Analyzing the Text

Before writing a rhetorical analysis, take the time to thoroughly analyze the text you'll be examining. This means more than just skimming through it; it requires a thorough understanding of its subtleties and complexities. Here are some questions to guide your analysis:

  • How does the text try to sway its audience? What methods does it use to convince or influence them?
  • Which rhetorical appeals—ethos (credibility), pathos (emotion), logos (logic)—does the author use, and how do they contribute to the overall argument?
  • What specific rhetorical devices and strategies does the author employ to effectively convey their message? Are there any patterns or recurring motifs?
  • How does the structure of the text contribute to its persuasive power or overall impact?
  • Are there any cultural, historical, or contextual factors that influence how the text is perceived or understood?

By scrutinizing the text in this manner, you'll gain a deeper understanding of how it functions and the techniques employed by the author to achieve their desired effect.

Rhetorical Analysis Introduction

The introduction sets the stage for your analysis by providing essential context and framing the discussion. Start by introducing the text you're analyzing, including the author's name and the title of the work. Provide some background information to give context to your analysis. For example, if you're analyzing a speech, mention the occasion or event where it was delivered.

Next, summarize the main arguments or claims made by the author. Highlight the rhetorical techniques they use to persuade their audience. Are they appealing to logic, emotion, credibility, or a combination of these? Use specific examples from the text to illustrate these techniques discussed by our dissertation service .

For instance, if you're analyzing a speech on climate change, mention the speaker's expertise in environmental science to establish credibility. Summarize the key points they make about the consequences of inaction and the urgent need for change.

Finally, conclude your introduction with a clear thesis statement. This statement should encapsulate the main argument or purpose of your analysis.

Rhetorical Analysis Body Paragraph

The body paragraphs form the crux of your analysis, where you delve into the details of the text and dissect its rhetorical strategies. Each paragraph should focus on a specific aspect of the text, such as the use of ethos, pathos, logos, or specific rhetorical devices.

Utilize Aristotle's rhetorical triangle and other key concepts introduced earlier to guide your analysis. Provide quotations or examples from the text to illustrate your points and explain why the author chose certain approaches. Evaluate the effectiveness of these strategies in achieving the author's goals and persuading the audience.

For instance, if you're discussing the use of pathos in a marketing campaign, analyze the emotional appeal of the imagery or language used and consider how it resonates with the target audience.

Rhetorical Analysis Conclusion

In the conclusion, it's crucial to reinforce your main arguments and evaluate the author's effectiveness in achieving their goals, whether you're writing an MLA or APA essay format . Reflect on the overall impact of the text on both its immediate audience and society at large, underscoring the importance of your analysis.

Resist the temptation to introduce new ideas in the conclusion. Instead, draw upon the points you've already explored in the body of your essay to strengthen your analysis. Conclude with a poignant statement that resonates with your readers, encapsulating the essence of your interpretation and leaving a lasting impression. This final remark should tie together the threads of your analysis, leaving the reader with a deeper understanding of the text's rhetorical strategies and significance.

Rhetorical Analysis Essay Example

In this section, you'll discover two essay samples that skillfully demonstrate the application of rhetorical analysis. These examples offer insightful insights into the effective use of rhetorical techniques in writing.

5 Rhetorical Analysis Essay Tips

Here are five focused tips that will help you lay a solid foundation for your examination.

  • Dissect Rhetorical Strategies : Break down the text to identify specific rhetorical devices such as metaphor, simile, or parallelism.
  • Evaluate Tone and Diction : Pay attention to the author's tone and word choice. Analyze how these elements contribute to the overall mood of the text.
  • Probe Ethos, Pathos, Logos : Explore how the author establishes credibility (ethos), evokes emotions (pathos), and employs logic (logos) to sway the audience.
  • Contextualize Historical Significance : Consider the historical, cultural, and social backdrop against which the text was written.
  • Craft a Structured Analysis : Organize your essay with a clear introduction, body paragraphs focusing on specific rhetorical elements, and a conclusion that synthesizes your findings.

Final Words

As we near the end, it's important to analyze carefully whether you're examining a speech, an advertisement, or a story. Pay attention to the smart tactics that influence our thinking. It's all about revealing how we communicate and relate to one another. Ultimately, understanding rhetoric offers a fresh perspective on the world beyond just academic success.

Looking to Take Your Academic Performance to the Next Level?

Say goodbye to stress, endless research, and sleepless nights - and hello to a brighter academic future.

What is a Rhetorical Analysis Essay?

How to structure a rhetorical analysis essay, how to write a rhetorical analysis essay, related articles.

How to Write an Analytical Essay

How to Write the AP Lang Rhetorical Analysis Essay (With Example)

November 27, 2023

how to write AP Lang rhetorical analysis essay example

Feeling intimidated by the AP Lang Rhetorical Analysis Essay? We’re here to help demystify. Whether you’re cramming for the AP Lang exam right now or planning to take the test down the road, we’ve got crucial rubric information, helpful tips, and an essay example to prepare you for the big day. This post will cover 1) What is the AP Lang Rhetorical Analysis Essay? 2) AP Lang Rhetorical Analysis Rubric 3) AP Lang Rhetorical Analysis: Sample Prompt 4) AP Lang Rhetorical Analysis Essay Example 5)AP Lang Rhetorical Analysis Essay Example: Why It Works

What is the AP Lang Rhetorical Analysis Essay?

The AP Lang Rhetorical Analysis Essay is one of three essays included in the written portion of the AP English Exam. The full AP English Exam is 3 hours and 15 minutes long, with the first 60 minutes dedicated to multiple-choice questions. Once you complete the multiple-choice section, you move on to three equally weighted essays that ask you to synthesize, analyze, and interpret texts and develop well-reasoned arguments. The three essays include:

Synthesis essay: You’ll review various pieces of evidence and then write an essay that synthesizes (aka combines and interprets) the evidence and presents a clear argument. Read our write up on How to Write the AP Lang Synthesis Essay here.

Argumentative essay: You’ll take a stance on a specific topic and argue your case.

Rhetorical essay: You’ll read a provided passage, then analyze the author’s rhetorical choices and develop an argument that explains why the author made those rhetorical choices.

AP Lang Rhetorical Analysis Rubric

The AP Lang Rhetorical Analysis Essay is graded on just 3 rubric categories: Thesis, Evidence and Commentary, and Sophistication . At a glance, the rubric categories may seem vague, but AP exam graders are actually looking for very particular things in each category. We’ll break it down with dos and don’ts for each rubric category:

Thesis (0-1 point)

There’s nothing nebulous when it comes to grading AP Lang Rhetorical Analysis Essay thesis. You either have one or you don’t. Including a thesis gets you one point closer to a high score and leaving it out means you miss out on one crucial point. So, what makes a thesis that counts?

  • Make sure your thesis argues something about the author’s rhetorical choices. Making an argument means taking a risk and offering your own interpretation of the provided text. This is an argument that someone else might disagree with.
  • A good test to see if you have a thesis that makes an argument. In your head, add the phrase “I think that…” to the beginning of your thesis. If what follows doesn’t logically flow after that phrase (aka if what follows isn’t something you and only you think), it’s likely you’re not making an argument.
  • Avoid a thesis that merely restates the prompt.
  • Avoid a thesis that summarizes the text but does not make an argument.

Evidence and Commentary (0-4 points)

This rubric category is graded on a scale of 0-4 where 4 is the highest grade. Per the AP Lang Rhetorical Analysis rubric, to get a 4, you’ll want to:

  • Include lots of specific evidence from the text. There is no set golden number of quotes to include, but you’ll want to make sure you’re incorporating more than a couple pieces of evidence that support your argument about the author’s rhetorical choices.
  • Make sure you include more than one type of evidence, too. Let’s say you’re working on your essay and have gathered examples of alliteration to include as supporting evidence. That’s just one type of rhetorical choice, and it’s hard to make a credible argument if you’re only looking at one type of evidence. To fix that issue, reread the text again looking for patterns in word choice and syntax, meaningful figurative language and imagery, literary devices, and other rhetorical choices, looking for additional types of evidence to support your argument.
  • After you include evidence, offer your own interpretation and explain how this evidence proves the point you make in your thesis.
  • Don’t summarize or speak generally about the author and the text. Everything you write must be backed up with evidence.
  • Don’t let quotes speak for themselves. After every piece of evidence you include, make sure to explain your interpretation. Also, connect the evidence to your overarching argument.

Sophistication (0-1 point)

In this case, sophistication isn’t about how many fancy vocabulary words or how many semicolons you use. According to College Board , one point can be awarded to AP Lang Rhetorical Analysis essays that “demonstrate sophistication of thought and/or a complex understanding of the rhetorical situation” in any of these three ways:

  • Explaining the significance or relevance of the writer’s rhetorical choices.
  • Explaining the purpose or function of the passage’s complexities or tensions.
  • Employing a style that is consistently vivid and persuasive.

Note that you don’t have to achieve all three to earn your sophistication point. A good way to think of this rubric category is to consider it a bonus point that you can earn for going above and beyond in depth of analysis or by writing an especially persuasive, clear, and well-structured essay. In order to earn this point, you’ll need to first do a good job with your thesis, evidence, and commentary.

  • Focus on nailing an argumentative thesis and multiple types of evidence. Getting these fundamentals of your essay right will set you up for achieving depth of analysis.
  • Explain how each piece of evidence connects to your thesis.
  • Spend a minute outlining your essay before you begin to ensure your essay flows in a clear and cohesive way.
  • Steer clear of generalizations about the author or text.
  • Don’t include arguments you can’t prove with evidence from the text.
  • Avoid complex sentences and fancy vocabulary words unless you use them often. Long, clunky sentences with imprecisely used words are hard to follow.

AP Lang Rhetorical Analysis: Sample Prompt

The sample prompt below is published online by College Board and is a real example from the 2021 AP Exam. The prompt provides background context, essay instructions, and the text you need to analyze. For sake of space, we’ve included the text as an image you can click to read. After the prompt, we provide a sample high scoring essay and then explain why this AP Lang Rhetorical Analysis essay example works.

Suggested time—40 minutes.

(This question counts as one-third of the total essay section score.)

On February 27, 2013, while in office, former president Barack Obama delivered the following address dedicating the Rosa Parks statue in the National Statuary Hall of the United States Capitol building. Rosa Parks was an African American civil rights activist who was arrested in 1955 for refusing to give up her seat on a segregated bus in Montgomery, Alabama. Read the passage carefully. Write an essay that analyzes the rhetorical choices Obama makes to convey his message.

In your response you should do the following:

  • Respond to the prompt with a thesis that analyzes the writer’s rhetorical choices.
  • Select and use evidence to support your line of reasoning.
  • Explain how the evidence supports your line of reasoning.
  • Demonstrate an understanding of the rhetorical situation.
  • Use appropriate grammar and punctuation in communicating your argument.

AP Lang Rhetorical Analysis Essay Example

In his speech delivered in 2013 at the dedication of Rosa Park’s statue, President Barack Obama acknowledges everything that Parks’ activism made possible in the United States. Telling the story of Parks’ life and achievements, Obama highlights the fact that Parks was a regular person whose actions accomplished enormous change during the civil rights era. Through the use of diction that portrays Parks as quiet and demure, long lists that emphasize the extent of her impacts, and Biblical references, Obama suggests that all of us are capable of achieving greater good, just as Parks did.

Although it might be a surprising way to start to his dedication, Obama begins his speech by telling us who Parks was not: “Rosa Parks held no elected office. She possessed no fortune” he explains in lines 1-2. Later, when he tells the story of the bus driver who threatened to have Parks arrested when she refused to get off the bus, he explains that Parks “simply replied, ‘You may do that’” (lines 22-23). Right away, he establishes that Parks was a regular person who did not hold a seat of power. Her protest on the bus was not part of a larger plan, it was a simple response. By emphasizing that Parks was not powerful, wealthy, or loud spoken, he implies that Parks’ style of activism is an everyday practice that all of us can aspire to.

AP Lang Rhetorical Analysis Essay Example (Continued)

Even though Obama portrays Parks as a demure person whose protest came “simply” and naturally, he shows the importance of her activism through long lists of ripple effects. When Parks challenged her arrest, Obama explains, Martin Luther King, Jr. stood with her and “so did thousands of Montgomery, Alabama commuters” (lines 27-28). They began a boycott that included “teachers and laborers, clergy and domestics, through rain and cold and sweltering heat, day after day, week after week, month after month, walking miles if they had to…” (lines 28-31). In this section of the speech, Obama’s sentences grow longer and he uses lists to show that Parks’ small action impacted and inspired many others to fight for change. Further, listing out how many days, weeks, and months the boycott lasted shows how Parks’ single act of protest sparked a much longer push for change.

To further illustrate Parks’ impact, Obama incorporates Biblical references that emphasize the importance of “that single moment on the bus” (lines 57-58). In lines 33-35, Obama explains that Parks and the other protestors are “driven by a solemn determination to affirm their God-given dignity” and he also compares their victory to the fall the “ancient walls of Jericho” (line 43). By of including these Biblical references, Obama suggests that Parks’ action on the bus did more than correct personal or political wrongs; it also corrected moral and spiritual wrongs. Although Parks had no political power or fortune, she was able to restore a moral balance in our world.

Toward the end of the speech, Obama states that change happens “not mainly through the exploits of the famous and the powerful, but through the countless acts of often anonymous courage and kindness” (lines 78-81). Through carefully chosen diction that portrays her as a quiet, regular person and through lists and Biblical references that highlight the huge impacts of her action, Obama illustrates exactly this point. He wants us to see that, just like Parks, the small and meek can change the world for the better.

AP Lang Rhetorical Analysis Essay Example: Why It Works

We would give the AP Lang Rhetorical Analysis essay above a score of 6 out of 6 because it fully satisfies the essay’s 3 rubric categories: Thesis, Evidence and Commentary, and Sophistication . Let’s break down what this student did:

The thesis of this essay appears in the last line of the first paragraph:

“ Through the use of diction that portrays Parks as quiet and demure, long lists that emphasize the extent of her impacts, and Biblical references, Obama suggests that all of us are capable of achieving greater good, just as Parks did .”

This student’s thesis works because they make a clear argument about Obama’s rhetorical choices. They 1) list the rhetorical choices that will be analyzed in the rest of the essay (the italicized text above) and 2) include an argument someone else might disagree with (the bolded text above).

Evidence and Commentary:

This student includes substantial evidence and commentary. Things they do right, per the AP Lang Rhetorical Analysis rubric:

  • They include lots of specific evidence from the text in the form of quotes.
  • They incorporate 3 different types of evidence (diction, long lists, Biblical references).
  • After including evidence, they offer an interpretation of what the evidence means and explain how the evidence contributes to their overarching argument (aka their thesis).

Sophistication

This essay achieves sophistication according to the AP Lang Rhetorical Analysis essay rubric in a few key ways:

  • This student provides an introduction that flows naturally into the topic their essay will discuss. Before they get to their thesis, they tell us that Obama portrays Parks as a “regular person” setting up their main argument: Obama wants all regular people to aspire to do good in the world just as Rosa Parks did.
  • They organize evidence and commentary in a clear and cohesive way. Each body paragraph focuses on just one type of evidence.
  • They explain how their evidence is significant. In the final sentence of each body paragraph, they draw a connection back to the overarching argument presented in the thesis.
  • All their evidence supports the argument presented in their thesis. There is no extraneous evidence or misleading detail.
  • They consider nuances in the text. Rather than taking the text at face value, they consider what Obama’s rhetorical choices imply and offer their own unique interpretation of those implications.
  • In their final paragraph, they come full circle, reiterate their thesis, and explain what Obama’s rhetorical choices communicate to readers.
  • Their sentences are clear and easy to read. There are no grammar errors or misused words.

AP Lang Rhetorical Analysis Essay—More Resources

Looking for more tips to help your master your AP Lang Rhetorical Analysis Essay? Brush up on 20 Rhetorical Devices High School Students Should Know and read our Tips for Improving Reading Comprehension . If you’re ready to start studying for another part of the AP English Exam, find more expert tips in our How to Write the AP Lang Synthesis blog post.

Considering what other AP classes to take? Read up on the Hardest AP Classes .

  • High School Success

' src=

Christina Wood

Christina Wood holds a BA in Literature & Writing from UC San Diego, an MFA in Creative Writing from Washington University in St. Louis, and is currently a Doctoral Candidate in English at the University of Georgia, where she teaches creative writing and first-year composition courses. Christina has published fiction and nonfiction in numerous publications, including The Paris Review , McSweeney’s , Granta , Virginia Quarterly Review , The Sewanee Review , Mississippi Review , and Puerto del Sol , among others. Her story “The Astronaut” won the 2018 Shirley Jackson Award for short fiction and received a “Distinguished Stories” mention in the 2019 Best American Short Stories anthology.

  • 2-Year Colleges
  • Application Strategies
  • Best Colleges by Major
  • Best Colleges by State
  • Big Picture
  • Career & Personality Assessment
  • College Essay
  • College Search/Knowledge
  • College Success
  • Costs & Financial Aid
  • Dental School Admissions
  • Extracurricular Activities
  • Graduate School Admissions
  • High Schools
  • Law School Admissions
  • Medical School Admissions
  • Navigating the Admissions Process
  • Online Learning
  • Private High School Spotlight
  • Summer Program Spotlight
  • Summer Programs
  • Test Prep Provider Spotlight

College Transitions Sidebar Block Image

“Innovative and invaluable…use this book as your college lifeline.”

— Lynn O'Shaughnessy

Nationally Recognized College Expert

College Planning in Your Inbox

Join our information-packed monthly newsletter.

I am a... Student Student Parent Counselor Educator Other First Name Last Name Email Address Zip Code Area of Interest Business Computer Science Engineering Fine/Performing Arts Humanities Mathematics STEM Pre-Med Psychology Social Studies/Sciences Submit

Rhetorical Analysis Sample Essay

Harriet Clark

Ms. Rebecca Winter

13 Feb. 2015

Not Quite a Clean Sweep: Rhetorical Strategies in

Grose's "Cleaning: The Final Feminist Frontier”

A woman’s work is never done: many American women grow up with this saying and feel it to be true. 1 One such woman, author Jessica Grose, wrote “Cleaning: The Final Feminist Frontier,” published in 2013 in the New Republic, 2 and she argues that while the men recently started taking on more of the childcare and cooking, cleaning still falls unfairly on women. 3 Grose begins building her credibility with personal facts and reputable sources, citing convincing facts and statistics, and successfully employing emotional appeals; however, toward the end of the article, her attempts to appeal to readers’ emotions weaken her credibility and ultimately, her argument. 4

In her article, Grose first sets the stage by describing a specific scenario of house-cleaning with her husband after being shut in during Hurricane Sandy, and then she outlines the uneven distribution of cleaning work in her marriage and draws a comparison to the larger feminist issue of who does the cleaning in a relationship. Grose continues by discussing some of the reasons that men do not contribute to cleaning: the praise for a clean house goes to the woman; advertising and media praise men’s cooking and childcare, but not cleaning; and lastly, it is just not fun. Possible solutions to the problem, Grose suggests, include making a chart of who does which chores, dividing up tasks based on skill and ability, accepting a dirtier home, and making cleaning more fun with gadgets. 5

Throughout her piece, Grose uses many strong sources that strengthen her credibility and appeal to ethos, as well as build her argument. 6 These sources include, “sociologists Judith Treas and Tsui-o Tai,” “a 2008 study from the University of New Hampshire,” and “P&G North America Fabric Care Brand Manager, Matthew Krehbiel” (qtd. in Grose). 7 Citing these sources boosts Grose’s credibility by showing that she has done her homework and has provided facts and statistics, as well as expert opinions to support her claim. She also uses personal examples from her own home life to introduce and support the issue, which shows that she has a personal stake in and first-hand experience with the problem. 8

Adding to her ethos appeals, Grose uses strong appeals to logos, with many facts and statistics and logical progressions of ideas. 9 She points out facts about her marriage and the distribution of household chores: “My husband and I both work. We split midnight baby feedings ...but ... he will admit that he’s never cleaned the bathroom, that I do the dishes nine times out of ten, and that he barely knows how the washer and dryer work in the apartment we’ve lived in for over eight months.” 10 These facts introduce and support the idea that Grose does more household chores than her husband. Grose continues with many statistics:

[A]bout 55 percent of American mothers employed full time do some housework on an average day, while only 18 percent of employed fathers do. ... [W]orking women with children are still doing a week and a half more of “second shift” work each year than their male partners. ... Even in the famously gender-neutral Sweden, women do 45 minutes more housework a day than their male partners. 11

These statistics are a few of many that logically support her claim that it is a substantial and real problem that men do not do their fair share of the chores. The details and numbers build an appeal to logos and impress upon the reader that this is a problem worth discussing. 12

Along with strong logos appeals, Grose effectively makes appeals to pathos in the beginning and middle sections. 13 Her introduction is full of emotionally-charged words and phrases that create a sympathetic image; Grose notes that she “was eight months pregnant” and her husband found it difficult to “fight with a massively pregnant person.” 14 The image she evokes of the challenges and vulnerabilities of being so pregnant, as well as the high emotions a woman feels at that time effectively introduce the argument and its seriousness. Her goal is to make the reader feel sympathy for her. Adding to this idea are words and phrases such as, “insisted,” “argued,” “not fun,” “sucks” “headachey,” “be judged,” “be shunned” (Grose). All of these words evoke negative emotions about cleaning, which makes the reader sympathize with women who feel “judged” and shunned”—very negative feelings. Another feeling Grose reinforces with her word choice is the concept of fairness: “fair share,” “a week and a half more of ‘second shift’ work,” “more housework,” “more gendered and less frequent.” These words help establish the unfairness that exists when women do all of the cleaning, and they are an appeal to pathos, or the readers’ feelings of frustration and anger with injustice. 15

However, the end of the article lacks the same level of effectiveness in the appeals to ethos. 16 For example, Grose notes that when men do housework, they are considered to be “’enacting “small instances of gender heroism,” or ‘SIGH’s’—which, barf.” 17 The usage of the word “barf” is jarring to the reader; unprofessional and immature, it is a shift from the researched, intelligent voice she has established and the reader is less likely to take the author seriously. This damages the strength of her credibility and her argument. 18

Additionally, her last statement in the article refers to her husband in a way that weakens the argument. 19 While returning to the introduction’s hook in the conclusion is a frequently-used strategy, Grose chooses to return to her discussion of her husband in a humorous way: Grose discusses solutions, and says there is “a huge, untapped market ... for toilet-scrubbing iPods. I bet my husband would buy one.” 20 Returning to her own marriage and husband is an appeal to ethos or personal credibility, and while that works well in the introduction, in the conclusion, it lacks the strength and seriousness that the topic deserves and was given earlier in the article. 21

Though Grose begins the essay by effectively persuading her readers of the unfair distribution of home-maintenance cleaning labor, she loses her power in the end, where she most needs to drive home her argument. Readers can see the problem exists in both her marriage and throughout the world; however, her shift to humor and sarcasm makes the reader not take the problem as seriously in the end. 22 Grose could have more seriously driven home the point that a woman’s work could be done: by a man. 23

Works Cited

Grose, Jessica. “Cleaning: The Final Feminist Frontier.” New Republic. The New Republic, 19 Mar. 2013. Web. 28 Mar. 2014.

  • Article author's claim or purpose
  • Summary of the article's main point in the second paragraph (could also be in the introduction)
  • Third paragraph begins with a transition and topic sentence that reflects the first topic in the thesis
  • Quotes illustrate how the author uses appeals to ethos
  • Analysis explains how the quotes show the effective use of ethos as noted in the thesis
  • Transition and topic sentence about the second point from the thesis
  • Quote that illustrates appeals to logos
  • Analysis explains how the quotes show the effective use of logos, as noted in the thesis
  • Transition and topic sentence about the third point from the thesis
  • Quotes that illustrate appeals to pathos
  • Analysis explains how the quotes show the effective use of pathos, as noted in the thesis
  • Transition and topic sentence about fourth point from the thesis
  • Quote illustrates how the author uses appeal to ethos
  • Analysis explains how quote supports thesis
  • Transition and topic sentence about fourth point from thesis
  • Conclusion returns to the ideas in the thesis and further develops them
  • Last sentence returns to the hook in the introduction

Learn more about the " Rhetorical Analysis Graphic Organizer ."

Learn more about " Pathos, Logos, and Ethos ."

Rhetorical Analysis Definition and Examples

The analysis can be used on any communication, even a bumper sticker

  • An Introduction to Punctuation

Sample Rhetorical Analyses

Examples and observations, analyzing effects, analyzing greeting card verse, analyzing starbucks, rhetorical analysis vs. literary criticism.

  • Ph.D., Rhetoric and English, University of Georgia
  • M.A., Modern English and American Literature, University of Leicester
  • B.A., English, State University of New York

Rhetorical analysis is a form of criticism or close reading that employs the principles of rhetoric to examine the interactions between a text, an author, and an audience . It's also called rhetorical criticism or pragmatic criticism.

Rhetorical analysis may be applied to virtually any text or image—a speech , an essay , an advertisement, a poem, a photograph, a web page, even a bumper sticker. When applied to a literary work, rhetorical analysis regards the work not as an aesthetic object but as an artistically structured instrument for communication. As Edward P.J. Corbett has observed, rhetorical analysis "is more interested in a literary work for what it does than for what it is."

  • A Rhetorical Analysis of Claude McKay's "Africa"
  • A Rhetorical Analysis of E.B. White's "The Ring of Time"
  • A Rhetorical Analysis of U2's "Sunday Bloody Sunday"
  • "Our response to the character of the author—whether it is called ethos, or 'implied author,' or style , or even tone—is part of our experience of his work, an experience of the voice within the masks, personae , of the work...Rhetorical criticism intensifies our sense of the dynamic relationships between the author as a real person and the more or less fictive person implied by the work." (Thomas O. Sloan, "Restoration of Rhetoric to Literary Study." The Speech Teacher )
  • "[R]hetorical criticism is a mode of analysis that focuses on the text itself. In that respect, it is like the practical criticism that the New Critics and the Chicago School indulge in. It is unlike these modes of criticism in that it does not remain inside the literary work but works outward from the text to considerations of the author and the audience...In talking about the ethical appeal in his 'Rhetoric,' Aristotle made the point that although a speaker may come before an audience with a certain antecedent reputation, his ethical appeal is exerted primarily by what he says in that particular speech before that particular audience. Likewise, in rhetorical criticism, we gain our impression of the author from what we can glean from the text itself—from looking at such things as his ideas and attitudes, his stance, his tone, his style. This reading back to the author is not the same sort of thing as the attempt to reconstruct the biography of a writer from his literary work. Rhetorical criticism seeks simply to ascertain the particular posture or image that the author is establishing in this particular work in order to produce a particular effect on a particular audience." (Edward P.J. Corbett, "Introduction" to " Rhetorical Analyses of Literary Works ")

"[A] complete   rhetorical analysis requires the researcher to move beyond identifying and labeling in that creating an inventory of the parts of a text represents only the starting point of the analyst's work. From the earliest examples of rhetorical analysis to the present, this analytical work has involved the analyst in interpreting the meaning of these textual components—both in isolation and in combination—for the person (or people) experiencing the text. This highly interpretive aspect of rhetorical analysis requires the analyst to address the effects of the different identified textual elements on the perception of the person experiencing the text. So, for example, the analyst might say that the presence of feature x will condition the reception of the text in a particular way. Most texts, of course, include multiple features, so this analytical work involves addressing the cumulative effects of the selected combination of features in the text." (Mark Zachry, "Rhetorical Analysis" from " The Handbook of Business Discourse , " Francesca Bargiela-Chiappini, editor)

"Perhaps the most pervasive type of repeated-word sentence used in greeting card verse is the sentence in which a word or group of words is repeated anywhere within the sentence, as in the following example:

In quiet and thoughtful ways , in happy and fun ways , all ways , and always , I love you.

In this sentence, the word ways is repeated at the end of two successive phrases, picked up again at the beginning of the next phrase, and then repeated as part of the word always . Similarly, the root word all initially appears in the phrase 'all ways' and is then repeated in a slightly different form in the homophonic word always . The movement is from the particular ('quiet and thoughtful ways,' 'happy and fun ways'), to the general ('all ways'), to the hyperbolic ('always')." (Frank D'Angelo, "The Rhetoric of Sentimental Greeting Card Verse." Rhetoric Review )

"Starbucks not just as an institution or as a set of verbal discourses or even advertising but as a material and physical site is deeply rhetorical...Starbucks weaves us directly into the cultural conditions of which it is constitutive. The color of the logo, the performative practices of ordering, making, and drinking the coffee, the conversations around the tables, and the whole host of other materialities and performances of/in Starbucks are at once the rhetorical claims and the enactment of the rhetorical action urged. In short, Starbucks draws together the tripartite relationships among place, body, and subjectivity. As a material/rhetorical place, Starbucks addresses and is the very site of a comforting and discomforting negotiation of these relationships." (Greg Dickinson, "Joe's Rhetoric: Finding Authenticity at Starbucks." Rhetoric Society Quarterly )

"What essentially are the differences between literary criticism analysis and rhetorical analysis? When a critic explicates Ezra Pound's Canto XLV , for example, and shows how Pound inveighs against usury as an offense against nature that corrupts society and the arts, the critic must point out the 'evidence'—the 'artistic proofs' of example and enthymeme [a formal syllogistic argument that is incompletely stated}—that Pound has drawn upon for his fulmination. The critic will also call attention to the 'arrangement' of the parts of that argument as a feature of the 'form' of the poem just as he may inquire into the language and syntax. Again these are matters that Aristotle assigned mainly to rhetoric...

"All critical essays dealing with the persona of a literary work are in reality studies of the 'Ethos' of the 'speaker' or 'narrator'—the voice—source of the rhythmic language which attracts and holds the kind of readers the poet desires as his audience, and the means this persona consciously or unconsciously chooses, in Kenneth Burke's term, to 'woo' that reader-audience." (Alexander Scharbach, "Rhetoric and Literary Criticism: Why Their Separation." College Composition and Communication )

  • Definition and Examples of Explication (Analysis)
  • Audience Analysis in Speech and Composition
  • Definition and Examples of Ethos in Classical Rhetoric
  • Stylistics and Elements of Style in Literature
  • A Rhetorical Analysis of U2's 'Sunday Bloody Sunday'
  • Definition of Belles-Lettres in English Grammer
  • Invented Ethos (Rhetoric)
  • What is a Rhetorical Situation?
  • Definition of Audience
  • Rhetoric: Definitions and Observations
  • Critical Analysis in Composition
  • Definition and Examples of the Topoi in Rhetoric
  • What Is the Second Persona?
  • What Is Phronesis?
  • Feminist Literary Criticism
  • Quotes About Close Reading

Rhetorical Analysis

Sample rhetorical analysis essay.

Student essay is used with permission.  It was originally submitted double-spaced with no extra spaces between the lines, featured proper MLA pagination, and 1/2″ paragraph indents.  The writing assignment asks for an argument about how several rhetorical elements work together to create a functioning whole in a given chapter of Michael Shermer’s 2004 book The Science of Good and Evil: Why People Cheat, Gossip, Care, Share, and Follow the Golden Rule .

Liana Monnat

English 101

Instructor: Joshua Dickinson

October 16, 2016

Michael Shermer Successfully Proves That Humans Can Be Good Without God

            In his chapter entitled “Can We Be Good Without God?” Michael Shermer’s objective is to prove that one does not need to be religious to be capable of moral behavior.  Shermer has, in his previous four chapters, taken care to establish ethos by demonstrating that he is an open-minded and intelligent fellow.  Judging by his use of vocabulary, he assumes his readers are also intelligent people, with whom he attempts to develop a connection through his intermittent use of humor.  Shermer has already proved that his arguments are well-supported by large quantities of evidence, which lets his audience know that what he is saying is inherently trustworthy.  Taking all of this into consideration and having carefully analyzed this chapter, the reader is compelled by logic to agree with Shermer that one can have religion without morality, and morality without religion.

Shermer begins his fifth chapter with an appeal to pathos.  He describes to readers the massacre perpetrated at Columbine High School by Eric Harris and Dylan Klebold (141).  His description of the event along with a photograph of the black-clad, angry-looking murderers gives the reader a glimpse of the terror that must have been experienced by those unfortunate enough to have been present at the massacre.  In building up to proving his argument, Shermer appeals to readers’ ability to reason by showing that outside influences do not cause a person to behave immorally.  He explains that in the aftermath of the event, many theories were put forth to rationalize the cause of Harris and Klebold’s murderous rampage.  Included in these causes were use of prescription drugs, cult or gang influence, a fatherless home, homosexuality, and exposure to violence in video games (143-144).  Shermer uses logic to point out that none of these causes were relevant, particularly the idea that video game violence may have been the cause.  He makes mention of several newspapers that make such a claim, but dismisses the articles as having been written by “wannabe social commentators” and “ad hoc social scientists” and lacking in evidence (143).  Shermer shows how ridiculous the notion of video games being the causal factor is by relaying testimony of other players of violent games.  They all point out that they have not been driven to violence by their gaming habits (143).  By presenting these testimonies, he appeals to our common sense and ability to reason as intelligent individuals to realize that if video games caused people to behave violently, all gamers who played violent games would exhibit violent behavior, which is certainly not the case.  Shermer has thus far proved to readers that outside influences do not cause a person to abandon their morality.

Having logically dismantled the previous cases, Shermer turns his focus to the subject of gun control.  He quips that those in favor of more gun control took advantage of the Columbine massacre by “squawking for more legislation” (146).  His use of the word ‘squawking’ brings chickens to mind, and the great amount of noise they produce at the slightest provocation.  I believe creating this visual was probably the intent behind his humorous choice of words.

Liberal gun control advocates thoroughly ridiculed, Shermer notes that conservatives answered the call for more gun control by insisting that guns were not the problem.  The problem, as conservatives saw it, was the evil souls of the people who used them to commit evil deeds (146).  I feel that Shermer purposefully saved mentioning the gun control issue for last because it deals with the ideas of evil, morality, and religion.  He has taken much care in the preceding chapters to make it clear that he does not believe that evil exists, and that morality is not a product of religion.  The issue of gun control seems a well-chosen topic from which to begin his argument of how morality is a thing separate from religion.

The first example of evidence Shermer offers in his argument is an excerpt from a letter read by Congressman Tom DeLay.  He uses the excerpt to bring to readers’ attention an argument that is commonly made to explain violent acts.  It implies that as science provides evidence for questions that people once looked to religion to answer, people no longer feel obligated by a higher power to behave morally (147).  Shermer disputes this argument by describing the case of another perpetrator of a school shooting.  Rumors of the perpetrator being an atheist were quickly dispelled by the family priest, whose explanation was that the boy was a sinner but not an atheist, to which Shermer sarcastically quips “Thank God for that” (147).  This remark demonstrates his disgust that the priest would imply that being a Christian murderer was less offensive than being an atheist.  With this evidence, Shermer has supported his argument and demonstrated to his audience that religious people do not necessarily have morals.

For Shermer’s next move, he takes into consideration the opinions of several credible people who believe that morality is impossible without religion.  He utilizes quotes from the 103rd archbishop of Canterbury, Pope Pious XI, and the deeply religious Dostoyevsky who all fervently insist that religion is absolutely necessary for morality (149-150).  Shermer then includes the religious views of Laura Schlessinger, his one-time colleague.  He immediately diminishes her religious credibility by referring to her as a “self-appointed religious authority” (150). This implies to readers that although she is considered a ‘religious authority’ her opinions should not be taken too seriously.   He points out that although Schlessinger claims to have grown up lacking morals due to an atheistic upbringing, she admits that her parents still managed to instill her with some degree of morality (151).  This admission helps support his idea that non-religious people can have morals, but is the only part of Shermer’s paragraphs about Schlessinger that appear to be relevant to his argument.  He continues on about her, however, and it becomes apparent to readers that Shermer once admired her work but was taken aback by her conversion to Judaism.  He further weakens her authority by poking fun at her, and readers (this reader, at least) cannot help but wonder if he only included these paragraphs about Schlessinger because he is still disgruntled about her defection from his cause.

Shermer has, through several quotes from religious authorities, demonstrated to his audience that religious people are adamant that religion is necessary for moral behavior.  In an effort to prove that they are wrong, he refutes the claims of these authorities by serving up examples of religious people that committed atrocities while zealously practicing their religion.  His go-to example is Hitler and the annihilation of the Jews in Germany.  He illustrates for readers the religious fervor of Hitler by quoting him as saying “I believe today that I am acting in the sense of the Almighty Creator.  By warding off the Jews I am fighting for the Lord’s work” (qtd. on 153).  By strategically using this quote, Shermer is proving to readers that not only did Hitler commit mass murder, he did so in the Lord’s name.  This example, more than any other, is meant to show that religion and morality are not related.

In perhaps his most convincing argument that one need not be religious to behave morally, Shermer gets personal.  He asks readers the question “What would you do if there was no God?” (154).  Now the reader must contemplate the point Shermer has been trying to make, but on an intimate level.  He forces one to admit that if it was learned that God did not exist, the vast majority of people would continue to behave morally.  Most people would not, free from fear of eternal reprisal, proceed to pillage, rape, and commit murder.  After this degree of self-examination, it would be illogical to disagree that morality is not a creation of religion.

In his chapter “Can We Be Good Without God?”, Shermer successfully proves that we can indeed be good without God.  He appealed to readers’ emotions by describing the nightmare that was the Columbine massacre and led his audience to logically conclude that no outside influences caused the perpetrators’ behavior.  Through the strategic use of quotes and examples, Shermer effectively demonstrated that contrary to the beliefs of religious authorities, deeply religious people are capable of behaving extremely immorally.  Shermer ingeniously substantiated his point by asking readers to ponder what their own behavior might be like without God holding them accountable for their actions.  I feel that this was his most convincing piece of evidence in support of his argument, it is hard to deny his logic when applying it to oneself.  It can be assumed that most readers would continue to behave morally, and would agree with Shermer that we can be good without God.

Shermer, Michael. “Can We Be Good Without God?”  The Science of Good and Evil: Why People Cheat,

Gossip, Care, Share, and Follow the Golden Rule.   Henry Holt and Company, LLC, 2004, pp.141-156.

  • Sample Rhetorical Analysis Essay (MLA Format). Authored by : Liana Monnat. Provided by : Jefferson Community College. Located at : http://www.sunyjefferson.edu . Project : ENG 101. License : CC BY-SA: Attribution-ShareAlike

9.5 Writing Process: Thinking Critically about Rhetoric

Learning outcomes.

By the end of this section, you will be able to:

  • Develop a rhetorical analysis through multiple drafts.
  • Identify and analyze rhetorical strategies in a rhetorical analysis.
  • Demonstrate flexible strategies for generating ideas, drafting, reviewing, collaborating, revising, rewriting, and editing.
  • Give and act on productive feedback for works in progress.

The ability to think critically about rhetoric is a skill you will use in many of your classes, in your work, and in your life to gain insight from the way a text is written and organized. You will often be asked to explain or to express an opinion about what someone else has communicated and how that person has done so, especially if you take an active interest in politics and government. Like Eliana Evans in the previous section, you will develop similar analyses of written works to help others understand how a writer or speaker may be trying to reach them.

Summary of Assignment: Rhetorical Analysis

The assignment is to write a rhetorical analysis of a piece of persuasive writing. It can be an editorial, a movie or book review, an essay, a chapter in a book, or a letter to the editor. For your rhetorical analysis, you will need to consider the rhetorical situation—subject, author, purpose, context, audience, and culture—and the strategies the author uses in creating the argument. Back up all your claims with evidence from the text. In preparing your analysis, consider these questions:

  • What is the subject? Be sure to distinguish what the piece is about.
  • Who is the writer, and what do you know about them? Be sure you know whether the writer is considered objective or has a particular agenda.
  • Who are the readers? What do you know or what can you find out about them as the particular audience to be addressed at this moment?
  • What is the purpose or aim of this work? What does the author hope to achieve?
  • What are the time/space/place considerations and influences of the writer? What can you know about the writer and the full context in which they are writing?
  • What specific techniques has the writer used to make their points? Are these techniques successful, unsuccessful, or questionable?

For this assignment, read the following opinion piece by Octavio Peterson, printed in his local newspaper. You may choose it as the text you will analyze, continuing the analysis on your own, or you may refer to it as a sample as you work on another text of your choosing. Your instructor may suggest presidential or other political speeches, which make good subjects for rhetorical analysis.

When you have read the piece by Peterson advocating for the need to continue teaching foreign languages in schools, reflect carefully on the impact the letter has had on you. You are not expected to agree or disagree with it. Instead, focus on the rhetoric—the way Peterson uses language to make his point and convince you of the validity of his argument.

Another Lens. Consider presenting your rhetorical analysis in a multimodal format. Use a blogging site or platform such as WordPress or Tumblr to explore the blogging genre, which includes video clips, images, hyperlinks, and other media to further your discussion. Because this genre is less formal than written text, your tone can be conversational. However, you still will be required to provide the same kind of analysis that you would in a traditional essay. The same materials will be at your disposal for making appeals to persuade your readers. Rhetorical analysis in a blog may be a new forum for the exchange of ideas that retains the basics of more formal communication. When you have completed your work, share it with a small group or the rest of the class. See Multimodal and Online Writing: Creative Interaction between Text and Image for more about creating a multimodal composition.

Quick Launch: Start with a Thesis Statement

After you have read this opinion piece, or another of your choice, several times and have a clear understanding of it as a piece of rhetoric, consider whether the writer has succeeded in being persuasive. You might find that in some ways they have and in others they have not. Then, with a clear understanding of your purpose—to analyze how the writer seeks to persuade—you can start framing a thesis statement : a declarative sentence that states the topic, the angle you are taking, and the aspects of the topic the rest of the paper will support.

Complete the following sentence frames as you prepare to start:

  • The subject of my rhetorical analysis is ________.
  • My goal is to ________, not necessarily to ________.
  • The writer’s main point is ________.
  • I believe the writer has succeeded (or not) because ________.
  • I believe the writer has succeeded in ________ (name the part or parts) but not in ________ (name the part or parts).
  • The writer’s strongest (or weakest) point is ________, which they present by ________.

Drafting: Text Evidence and Analysis of Effect

As you begin to draft your rhetorical analysis, remember that you are giving your opinion on the author’s use of language. For example, Peterson has made a decision about the teaching of foreign languages, something readers of the newspaper might have different views on. In other words, there is room for debate and persuasion.

The context of the situation in which Peterson finds himself may well be more complex than he discusses. In the same way, the context of the piece you choose to analyze may also be more complex. For example, perhaps Greendale is facing an economic crisis and must pare its budget for educational spending and public works. It’s also possible that elected officials have made budget cuts for education a part of their platform or that school buildings have been found obsolete for safety measures. On the other hand, maybe a foreign company will come to town only if more Spanish speakers can be found locally. These factors would play a part in a real situation, and rhetoric would reflect that. If applicable, consider such possibilities regarding the subject of your analysis. Here, however, these factors are unknown and thus do not enter into the analysis.

Introduction

One effective way to begin a rhetorical analysis is by using an anecdote, as Eliana Evans has done. For a rhetorical analysis of the opinion piece, a writer might consider an anecdote about a person who was in a situation in which knowing another language was important or not important. If they begin with an anecdote, the next part of the introduction should contain the following information:

  • Author’s name and position, or other qualification to establish ethos
  • Title of work and genre
  • Author’s thesis statement or stance taken (“Peterson argues that . . .”)
  • Brief introductory explanation of how the author develops and supports the thesis or stance
  • If relevant, a brief summary of context and culture

Once the context and situation for the analysis are clear, move directly to your thesis statement. In this case, your thesis statement will be your opinion of how successful the author has been in achieving the established goal through the use of rhetorical strategies. Read the sentences in Table 9.1 , and decide which would make the best thesis statement. Explain your reasoning in the right-hand column of this or a similar chart.

The introductory paragraph or paragraphs should serve to move the reader into the body of the analysis and signal what will follow.

Your next step is to start supporting your thesis statement—that is, how Octavio Peterson, or the writer of your choice, does or does not succeed in persuading readers. To accomplish this purpose, you need to look closely at the rhetorical strategies the writer uses.

First, list the rhetorical strategies you notice while reading the text, and note where they appear. Keep in mind that you do not need to include every strategy the text contains, only those essential ones that emphasize or support the central argument and those that may seem fallacious. You may add other strategies as well. The first example in Table 9.2 has been filled in.

When you have completed your list, consider how to structure your analysis. You will have to decide which of the writer’s statements are most effective. The strongest point would be a good place to begin; conversely, you could begin with the writer’s weakest point if that suits your purposes better. The most obvious organizational structure is one of the following:

  • Go through the composition paragraph by paragraph and analyze its rhetorical content, focusing on the strategies that support the writer’s thesis statement.
  • Address key rhetorical strategies individually, and show how the author has used them.

As you read the next few paragraphs, consult Table 9.3 for a visual plan of your rhetorical analysis. Your first body paragraph is the first of the analytical paragraphs. Here, too, you have options for organizing. You might begin by stating the writer’s strongest point. For example, you could emphasize that Peterson appeals to ethos by speaking personally to readers as fellow citizens and providing his credentials to establish credibility as someone trustworthy with their interests at heart.

Following this point, your next one can focus, for instance, on Peterson’s view that cutting foreign language instruction is a danger to the education of Greendale’s children. The points that follow support this argument, and you can track his rhetoric as he does so.

You may then use the second or third body paragraph, connected by a transition, to discuss Peterson’s appeal to logos. One possible transition might read, “To back up his assertion that omitting foreign languages is detrimental to education, Peterson provides examples and statistics.” Locate examples and quotes from the text as needed. You can discuss how, in citing these statistics, Peterson uses logos as a key rhetorical strategy.

In another paragraph, focus on other rhetorical elements, such as parallelism, repetition, and rhetorical questions. Moreover, be sure to indicate whether the writer acknowledges counterclaims and whether they are accepted or ultimately rejected.

The question of other factors at work in Greendale regarding finances, or similar factors in another setting, may be useful to mention here if they exist. As you continue, however, keep returning to your list of rhetorical strategies and explaining them. Even if some appear less important, they should be noted to show that you recognize how the writer is using language. You will likely have a minimum of four body paragraphs, but you may well have six or seven or even more, depending on the work you are analyzing.

In your final body paragraph, you might discuss the argument that Peterson, for example, has made by appealing to readers’ emotions. His calls for solidarity at the end of the letter provide a possible solution to his concern that the foreign language curriculum “might vanish like a puff of smoke.”

Use Table 9.3 to organize your rhetorical analysis. Be sure that each paragraph has a topic sentence and that you use transitions to flow smoothly from one idea to the next.

As you conclude your essay, your own logic in discussing the writer’s argument will make it clear whether you have found their claims convincing. Your opinion, as framed in your conclusion, may restate your thesis statement in different words, or you may choose to reveal your thesis at this point. The real function of the conclusion is to confirm your evaluation and show that you understand the use of the language and the effectiveness of the argument.

In your analysis, note that objections could be raised because Peterson, for example, speaks only for himself. You may speculate about whether the next edition of the newspaper will feature an opposing opinion piece from someone who disagrees. However, it is not necessary to provide answers to questions you raise here. Your conclusion should summarize briefly how the writer has made, or failed to make, a forceful argument that may require further debate.

For more guidance on writing a rhetorical analysis, visit the Illinois Writers Workshop website or watch this tutorial .

Peer Review: Guidelines toward Revision and the “Golden Rule”

Now that you have a working draft, your next step is to engage in peer review, an important part of the writing process. Often, others can identify things you have missed or can ask you to clarify statements that may be clear to you but not to others. For your peer review, follow these steps and make use of Table 9.4 .

  • Quickly skim through your peer’s rhetorical analysis draft once, and then ask yourself, What is the main point or argument of my peer’s work?
  • Highlight, underline, or otherwise make note of statements or instances in the paper where you think your peer has made their main point.
  • Look at the draft again, this time reading it closely.
  • Ask yourself the following questions, and comment on the peer review sheet as shown.

The Golden Rule

An important part of the peer review process is to keep in mind the familiar wisdom of the “Golden Rule”: treat others as you would have them treat you. This foundational approach to human relations extends to commenting on others’ work. Like your peers, you are in the same situation of needing opinion and guidance. Whatever you have written will seem satisfactory or better to you because you have written it and know what you mean to say.

However, your peers have the advantage of distance from the work you have written and can see it through their own eyes. Likewise, if you approach your peer’s work fairly and free of personal bias, you’re likely to be more constructive in finding parts of their writing that need revision. Most important, though, is to make suggestions tactfully and considerately, in the spirit of helping, not degrading someone’s work. You and your peers may be reluctant to share your work, but if everyone approaches the review process with these ideas in mind, everyone will benefit from the opportunity to provide and act on sincerely offered suggestions.

Revising: Staying Open to Feedback and Working with It

Once the peer review process is complete, your next step is to revise the first draft by incorporating suggestions and making changes on your own. Consider some of these potential issues when incorporating peers’ revisions and rethinking your own work.

  • Too much summarizing rather than analyzing
  • Too much informal language or an unintentional mix of casual and formal language
  • Too few, too many, or inappropriate transitions
  • Illogical or unclear sequence of information
  • Insufficient evidence to support main ideas effectively
  • Too many generalities rather than specific facts, maybe from trying to do too much in too little time

In any case, revising a draft is a necessary step to produce a final work. Rarely will even a professional writer arrive at the best point in a single draft. In other words, it’s seldom a problem if your first draft needs refocusing. However, it may become a problem if you don’t address it. The best way to shape a wandering piece of writing is to return to it, reread it, slow it down, take it apart, and build it back up again. Approach first-draft writing for what it is: a warm-up or rehearsal for a final performance.

Suggestions for Revising

When revising, be sure your thesis statement is clear and fulfills your purpose. Verify that you have abundant supporting evidence and that details are consistently on topic and relevant to your position. Just before arriving at the conclusion, be sure you have prepared a logical ending. The concluding statement should be strong and should not present any new points. Rather, it should grow out of what has already been said and return, in some degree, to the thesis statement. In the example of Octavio Peterson, his purpose was to persuade readers that teaching foreign languages in schools in Greendale should continue; therefore, the conclusion can confirm that Peterson achieved, did not achieve, or partially achieved his aim.

When revising, make sure the larger elements of the piece are as you want them to be before you revise individual sentences and make smaller changes. If you make small changes first, they might not fit well with the big picture later on.

One approach to big-picture revising is to check the organization as you move from paragraph to paragraph. You can list each paragraph and check that its content relates to the purpose and thesis statement. Each paragraph should have one main point and be self-contained in showing how the rhetorical devices used in the text strengthen (or fail to strengthen) the argument and the writer’s ability to persuade. Be sure your paragraphs flow logically from one to the other without distracting gaps or inconsistencies.

As an Amazon Associate we earn from qualifying purchases.

This book may not be used in the training of large language models or otherwise be ingested into large language models or generative AI offerings without OpenStax's permission.

Want to cite, share, or modify this book? This book uses the Creative Commons Attribution License and you must attribute OpenStax.

Access for free at https://openstax.org/books/writing-guide/pages/1-unit-introduction
  • Authors: Michelle Bachelor Robinson, Maria Jerskey, featuring Toby Fulwiler
  • Publisher/website: OpenStax
  • Book title: Writing Guide with Handbook
  • Publication date: Dec 21, 2021
  • Location: Houston, Texas
  • Book URL: https://openstax.org/books/writing-guide/pages/1-unit-introduction
  • Section URL: https://openstax.org/books/writing-guide/pages/9-5-writing-process-thinking-critically-about-rhetoric

© Dec 19, 2023 OpenStax. Textbook content produced by OpenStax is licensed under a Creative Commons Attribution License . The OpenStax name, OpenStax logo, OpenStax book covers, OpenStax CNX name, and OpenStax CNX logo are not subject to the Creative Commons license and may not be reproduced without the prior and express written consent of Rice University.

Rhetorical Analysis Essay

Rhetorical Analysis Essay Example

Nova A.

Rhetorical Analysis Essay Example - Free Samples

11 min read

Rhetorical Analysis Essay Example

People also read

Rhetorical Analysis Essay - A Complete Guide With Examples

Rhetorical Analysis Essay Topics – 120+ Unique Ideas

Crafting an Effective Rhetorical Analysis Essay Outline - Free Samples!

Ethos, Pathos, and Logos - Structure, Usage & Examples

Writing a rhetorical analysis essay for academics can be really demanding for students. This type of paper requires high-level analyzing abilities and professional writing skills to be drafted effectively.

As this essay persuades the audience, it is essential to know how to take a strong stance and develop a thesis. 

This article will find some examples that will help you with your rhetorical analysis essay writing effortlessly. 

Arrow Down

  • 1. Good Rhetorical Analysis Essay Example
  • 2. Rhetorical Analysis Essay Example AP Lang 2023
  • 3. Rhetorical Analysis Essay Examples for Students 
  • 4. Writing a Visual Rhetorical Analysis Essay with Example 
  • 5. Rhetorical Analysis Essay Writing Tips

Good Rhetorical Analysis Essay Example

The step-by-step writing process of a rhetorical analysis essay is far more complicated than ordinary academic essays. This essay type critically analyzes the rhetorical means used to persuade the audience and their efficiency. 

The example provided below is the best rhetorical analysis essay example:

Rhetorical Analysis Essay Sample

In this essay type, the author uses rhetorical approaches such as ethos, pathos, and logos .  These approaches are then studied and analyzed deeply by the essay writers to weigh their effectiveness in delivering the message.

Let’s take a look at the following example to get a better idea;

The outline and structure of a rhetorical analysis essay are important. 

According to the essay outline, the essay is divided into three sections: 

  • Introduction
  • Ethos 
  • Logos 

A rhetorical analysis essay outline is the same as the traditional one. The different parts of the rhetorical analysis essay are written in the following way:

Rhetorical Analysis Introduction Example

The introductory paragraph of a rhetorical analysis essay is written for the following purpose:

  • To provide basic background information about the chosen author and the text.
  • Identify the target audience of the essay. 

An introduction for a rhetorical essay is drafted by:

  • Stating an opening sentence known as the hook statement. This catchy sentence is prepared to grab the audience’s attention to the paper. 
  • After the opening sentence, the background information of the author and the original text are provided. 

For example, a rhetorical analysis essay written by Lee Jennings on“The Right Stuff” by David Suzuki. Lee started the essay by providing the introduction in the following way:

Analysis of the Example: 

  • Suzuki stresses the importance of high school education. He prepares his readers for a proposal to make that education as valuable as possible.
  • A rhetorical analysis can show how successful Suzuki was in using logos, pathos, and ethos. He had a strong ethos because of his reputation. 
  • He also used pathos to appeal to parents and educators. However, his use of logos could have been more successful.
  • Here Jennings stated the background information about the text and highlighted the rhetorical techniques used and their effectiveness. 

Thesis Statement Example for Rhetorical Analysis Essay 

A thesis statement of a rhetorical analysis essay is the writer’s stance on the original text. It is the argument that a writer holds and proves it using the evidence from the original text. 

A thesis statement for a rhetorical essay is written by analyzing the following elements of the original text:

  • Diction - It refers to the author’s choice of words and the tone
  • Imagery - The visual descriptive language that the author used in the content. 
  • Simile - The comparison of things and ideas

In Jennings's analysis of “The Right Stuff,” the thesis statement was:

Example For Rhetorical Analysis Thesis Statement

Rhetorical Analysis Body Paragraph Example 

In the body paragraphs of your rhetorical analysis essay, you dissect the author's work, analyze their use of rhetorical techniques, and provide evidence to support your analysis. 

Let's look at an example that analyzes the use of ethos in David Suzuki's essay:

Rhetorical Analysis Conclusion Example

All the body paragraphs lead the audience towards the conclusion.

For example, the conclusion of “The Right Stuff” is written in the following way by Jennings:

In the conclusion section, Jennings summarized the major points and restated the thesis statement to prove them. 

Rhetorical Essay Example For The Right Stuff by David Suzuki

Rhetorical Analysis Essay Example AP Lang 2023

Writing a rhetorical analysis for the AP Language and Composition course can be challenging. So drafting it correctly is important to earn good grades. 

To make your essay effective and winning, follow the tips provided by professionals below:

Step #1: Understand the Prompt

Understanding the prompt is the first thing to produce an influential rhetorical paper. It is mandatory for this academic writing to read and understand the prompt to know what the task demands from you. 

Step #2: Stick to the Format

The content for the rhetorical analysis should be appropriately organized and structured. For this purpose, a proper outline is drafted. 

The rhetorical analysis essay outline divides all the information into different sections, such as the introduction, body, and conclusion.  The introduction should explicitly state the background information and the thesis statement. 

All the body paragraphs should start with a topic sentence to convey a claim to the readers. Provide a thorough analysis of these claims in the paragraph to support your topic sentence. 

Step #3: Use Rhetorical Elements to Form an Argument 

Analyze the following things in the text to form an argument for your essay:

  • Language (tone and words)
  • Organizational structure
  • Rhetorical Appeals ( ethos, pathos, and logos) 

Once you have analyzed the rhetorical appeals and other devices like imagery and diction, you can form a strong thesis statement. The thesis statement will be the foundation on which your essay will be standing. 

AP Language Rhetorical Essay Sample

AP Rhetorical Analysis Essay Template

Rhetorical Analysis Essay Example AP Lang

AP Lang Rhetorical Analysis Essay Example

Rhetorical Analysis Essay Examples for Students 

Here are a few more examples to help the students write a rhetorical analysis essay:

Rhetorical Analysis Essay Example Ethos, Pathos, Logos

Rhetorical Analysis Essay Example Outline

Rhetorical Analysis Essay Example College

Rhetorical Analysis Essay Example APA Format

Compare and Contrast Rhetorical Analysis Essay Example

Comparative Rhetorical Analysis Essay Example

How to Start Rhetorical Analysis Essay Example

Rhetorical Analysis Essay Example High School

Rhetorical Analysis Essay Example APA Sample

Rhetorical Analysis Essay Example Of a Song

Florence Kelley Speech Rhetorical Analysis Essay Example

Rhetorical Analysis Essay Example MLA

Writing a Visual Rhetorical Analysis Essay with Example 

The visual rhetorical analysis essay determines how pictures and images communicate messages and persuade the audience. 

Usually, visual rhetorical analysis papers are written for advertisements. This is because they use strong images to convince the audience to behave in a certain way. 

To draft a perfect visual rhetorical analysis essay, follow the tips below:

  • Analyze the advertisement deeply and note every minor detail. 
  • Notice objects and colors used in the image to gather every detail.
  • Determine the importance of the colors and objects and analyze why the advertiser chose the particular picture. 
  • See what you feel about the image.
  • Consider the objective of the image. Identify the message that the image is portraying. 
  • Identify the targeted audience and how they respond to the picture. 

An example is provided below to give students a better idea of the concept. 

Simplicity Breeds Clarity Visual Rhetorical Analysis Essay Example

Rhetorical Analysis Essay Writing Tips

Follow the tips provided below to make your rhetorical writing compelling. 

  • Choose an engaging topic for your essay. The rhetorical analysis essay topic should be engaging to grab the reader’s attention.
  • Thoroughly read the original text.
  • Identify the SOAPSTone. From the text, determine the speaker, occasions, audience, purpose, subject, and tone.
  • Develop a thesis statement to state your claim over the text.
  • Draft a rhetorical analysis essay outline.
  • Write an engaging essay introduction by giving a hook statement and background information. At the end of the introductory paragraph, state the thesis statement.
  • The body paragraphs of the rhetorical essay should have a topic sentence. Also, in the paragraph, a thorough analysis should be presented.
  • For writing a satisfactory rhetorical essay conclusion, restate the thesis statement and summarize the main points.
  • Proofread your essay to check for mistakes in the content. Make your edits before submitting the draft.

Following the tips and the essay's correct writing procedure will guarantee success in your academics. 

We have given you plenty of examples of a rhetorical analysis essay. But if you are still struggling to draft a great rhetorical analysis essay, it is suggested to take a professional’s help.

MyPerfectWords.com can assist you with all your academic assignments. The top essay writer service that we provide is reliable. If you are confused about your writing assignments and have difficulty meeting the deadline, get help from custom essay writing online .

Hire our analytical essay writing service today at the most reasonable prices. 

AI Essay Bot

Write Essay Within 60 Seconds!

Nova A.

Nova Allison is a Digital Content Strategist with over eight years of experience. Nova has also worked as a technical and scientific writer. She is majorly involved in developing and reviewing online content plans that engage and resonate with audiences. Nova has a passion for writing that engages and informs her readers.

Get Help

Paper Due? Why Suffer? That’s our Job!

Keep reading

Rhetorical Analysis Essay

Logo for Pressbooks @ Howard Community College

Want to create or adapt books like this? Learn more about how Pressbooks supports open publishing practices.

17 Rhetorical Modes for Paragraphs & Essays

Questions to Ponder

Before you read this chapter, discuss with partners:

  • What are rhetorical modes (also called “patterns of organization” and “methods of development”)? Can you list some examples?
  • Why are rhetorical modes important in writing? Jot down your ideas.

purple flowers in pattern

Now read the graphic below. Can you add to the list of rhetorical modes that you created with your partners?

Flow Chart. Central idea: Choosing Paragraph Patterns. Radiating from top right: Narration - introduction, to tell a story that makes a point, to give background on people or event, to show sequence of events. Process - to show steps of action, to explain how to do something. Example/Illustration - to clarify a point or concept, to give a picture or specific instance, to make the abstract real. Analogy - to compare scenarios, to compare to a settled outcome, to compare one event to another very different one. Definition - to clarify meaning, to set foundation of argument, to give background. Comparison/contrast - to draw distinction between items, to find common ground. Description - to give details, to create a picture. Cause/effect - to lead from one item to another, to argue logic of evidence of action. Classification/Division - to put items in categories, to clarify comparison of items in a category, to divide items by characteristics.

Rhetorical Modes

Rhetorical modes are also called patterns of organization or methods of development ; they are the ways that authors and speakers organize their ideas to communicate effectively. The rhetorical modes that are covered here are best used as ways to look at what’s already happening in your draft and to consider how you might emphasize or expand on any existing patterns. You might already be familiar with some of these patterns because instructors will sometimes assign them as the purpose for writing an essay. For example, you might have been asked to write a cause and effect essay or a comparison and contrast essay.

Patterns of organization or methods of developing content usually happen naturally as a consequence of the way the writer engages with and organizes information while writing. That is to say, most writers don’t sit down and say, “I think I’ll write a cause and effect essay today.”  Instead, a writer might be more likely to be interested in a topic, say, the state of drinking water in the local community, and as the writer begins to explore the topic, certain cause and effect relationships between environmental pollutants and the community water supply may begin to emerge . And in fact, many times, one essay may incorporate two or more rhetorical modes, as the author makes an argument for their point of view.

Activity A ~ Brainstorming Rhetorical Modes

Pause here to brainstorm ideas with your partner. Using the chart above (“ Choosing Paragraph Patterns “), discuss some of the topics below. Which mode(s) might you use in an essay about these topics? Would you need to explore more than one rhetorical mode for each topic?

  • Gender roles
  • Race in America
  • The value of art in society
  • Travel as part of a well-rounded education
  • Drugs and alcohol
  • Advice to new parents
  • Advice to teachers
  • The value of making mistakes
  • How you’d spend a million dollars
  • What a tough day at work taught you about yourself or others
  • My family history
  • Your idea: ___________

Keep reading to consider some of the ways that these strategies can help you as you revise a draft.

Cause/Effect

Do you see a potential cause-and-effect relationship developing in your draft?  The cause/effect pattern may be used to identify one or more causes followed by one or more effects or results. Or you may reverse this sequence and describe effects first and then the cause or causes. For example, the causes of water pollution might be followed by its effects on both humans and animals. Use the signal words cause ,  effect , and  result , to cue the reader about your about the relationships that you’re establishing.

Here’s an example article from T he New York Times , “ Rough Times Take Bloom Off a New Year’s Rite, the Rose Parade ,” that explores the cause and effect relationship (from 2011) between Pasadena’s budgetary challenges and the ability of their Rose Parade floats to deck themselves out in full bloom.

Problem/Solution

At some point does your essay explore a problem or suggest a solution? The problem/solution pattern is commonly used in identifying something that’s wrong and in contemplating what might be done to remedy the situation. For example, the problem of water pollution could be described, followed by ideas of new ways to solve the problem. There are probably more ways to organize a problem/solution approach, but here are three possibilities:

  • Describe the problem, followed by the solution
  • Propose the solution first and then describe the problems that motivated it
  • Explain a problem, followed by several solutions, and select one solution as the best

Emphasize the words problem  and  solution  to signal these sections of your paper for your reader.

Here’s an example article from T he New York Times , “ Monks Embrace Web to Reach Recruits ,” that highlights an unexpected approach by a group of Benedictine monks in Rhode Island; they’ve turned to social media to grow their dwindling membership.

Compare/Contrast

Are you trying to define something? Do you need your readers to understand what something is and what it is not? The compare-and-contrast method of development is particularly useful in extending a definition, or anywhere you need to show how a subject is like or unlike another subject. For example, the statement is often made that drug abuse is a medical problem instead of a criminal justice issue. An author might attempt to prove this point by comparing drug addiction to AIDS, cancer, or heart disease to redefine the term “addiction” as a medical problem. A statement in opposition to this idea could just as easily establish contrast by explaining all the ways that addiction is different from what we traditionally understand as an illness. In seeking to establish comparison or contrast in your writing, some words or terms that might be useful are by contrast ,  in comparison ,  while ,  some , and  others .

Here’s an example article from T he New York Times “ Who Wants to Shop in a Big Box Store, Anyway? ” The author explores some interesting differences between the average American and average Indian consumer to contemplate the potential success of big box stores in India and also to contemplate why these giant big box corporations, like Walmart or Target, might have to rethink their business model.

yellow umbrella on surface of water at daytime

These three methods of development—cause/effect, problem/solution, and compare/contrast—are just a few ways to organize and develop ideas and content in your essays. It’s important to note that they should not be a starting point for writers who want to write something authentic, to discuss something that they care deeply about. Instead, they can be a great way to help you look for what’s already happening with your topic or in a draft, to help you to write more, or to help you reorganize some parts of an essay that seem to lack connection or feel disjointed.

Sometimes writers incorporate a variety of modes in any one essay. For example, under the umbrella of an argument essay, and author might choose to write paragraphs showing cause and effect, description, and narrative. The rhetorical mode writers choose depends on the purpose for writing. Rhetorical modes are a set of tools that will give you greater flexibility and effectiveness in communicating with your audience and expressing ideas.

In addition to cause/effect , problem/solution , and compare/contrast , there are many other types of rhetorical modes:

  • Classification and division , often used in science, takes large ideas and divides them into manageable chunks of information, classifying and organizing them into types and parts.
  • Definition  clarifies the meaning of terms and concepts, providing context and description for deeper understanding of those ideas.
  • Description  provides detailed information using adjectives that appeal to the five senses (what people see, hear, smell, taste, and touch) as well as other vivid details that help readers visualize or understand an item or concept.
  • Evaluation  analyzes and judges the value and merit of an essay, a concept, or topic.
  • Illustration  provides examples and evidence in detail to support, explain, and analyze a main point or idea.
  • Narrative  uses fictional or nonfictional stories in a chronological sequence of events, often including detailed descriptions and appeals to the senses and emotions of readers while storytelling to reveal a theme or moment.
  • Persuasion  (i.e., argumentation) logically attempts to convince readers to agree with an opinion or take an action; the argument also acknowledges opposing viewpoints and accommodates and/or refutes them with diplomatic and respectful language, as well as provides precise and accurate evidence and other expert supporting details.
  • Process analysis  describes and explains, step by step, chronologically, in detail, and with precision and accuracy, how to do something or how something works.

Assignment prompts for college essays may require a specific rhetorical mode, or you may be able to choose the best mode(s) to express your ideas clearly. Either way, be sure to ask your instructor if you are not sure which rhetorical mode(s) to use.

Key Takeaways

Why are rhetorical modes important?

  • As readers, understanding an author’s rhetorical mode helps us to understand the text, and to read and think critically.
  • Knowing the rhetorical mode helps us to identify the author’s main ideas, which helps us to summarize the author’s work.
  • As writers, we use rhetorical modes to make our writing clearer; they help us signal our topic and direction to our readers.
  • Rhetorical modes also help us to develop support and keep our readers interested.

Activity B ~ Identifying Rhetorical Modes

  • Read a printed or online essay or article. A letter to the editor or an editorial from a newspaper would be perfect. Then, with a partner, identify the modes of writing found in the article. (Use the lists above to help.) Analyze the different choices the writer has made about language and organization to express a point of view. Notice how the author may combine rhetorical modes (for example, a problem-solution article that uses cause-and-effect organization in some paragraphs, or a definition pattern that uses narrative or compare and contrast paragraphs to develop similarities or differences).
  • Select, read, and annotate a sample student essay in a specific style as provided in “ Readings: Examples of Essays ” from Saylor Academy . Note in the margins or on another sheet of paper what rhetorical mode each paragraph uses, how those modes and paragraphs support the overall rhetorical mode of the essay, and whether each paragraph does so successfully or not. Discuss in small groups and summarize your findings to report to the rest of the class.

If you want to learn more about three common rhetorical modes, read what the New York Times  has to say in their learning blog article, “ Compare-Contrast, Cause-Effect, Problem Solution: Common ‘Text Types’ in The Times .”

Note: links open in new tabs.

This chapter was modified from the following Open Educational Resources:

“Patterns of Organization and Methods of Development ” from The Word on College Reading and Writing by Carol Burnell, Jaime Wood, Monique Babin, Susan Pesznecker, and Nicole Rosevear, which is licensed under a Creative Commons Attribution-NonCommercial 4.0 International License.

“ Introduction ” from  English Composition   by Karyl Garland, Ann Inoshita, Jeanne K. Tsutsui Keuma, Kate Sims, and Tasha Williams, is licensed under a  Creative Commons Attribution 4.0 International License

“ Chapter 10: The Rhetorical Modes ” and “ Chapter 15: Readings: Examples of Essays ,” from  Writing for Success from Saylor Academy, which is licensed under Creative Commons Attribution-NonCommercial-ShareAlike 3.0.

CC BY-NC-SA 3.0

to think about

to write quickly

to come out, to be revealed

to decorate themselves

to fix; to make right

getting smaller

ENGLISH 087: Academic Advanced Writing Copyright © 2020 by Nancy Hutchison is licensed under a Creative Commons Attribution-NonCommercial-ShareAlike 4.0 International License , except where otherwise noted.

Share This Book

Logo for Pressbooks@MSL

Chapter 6: Thinking and Analyzing Rhetorically

6.4 Rhetorical Appeals: Logos, Pathos, and Ethos Defined

Melanie Gagich & Emilie Zickel

Rhetoric, as the previous chapters have discussed, is the way that authors use and manipulate language in order to persuade an audience. Once we understand the rhetorical situation out of which a text is created (why it was written, for whom it was written, by whom it was written, how the medium in which it was written creates certain constraints, or perhaps freedoms of expression), we can look at how all of those contextual elements shape the author’s creation of the text.

We can look first at the classical rhetorical appeals, which are the three ways to classify authors’ intellectual, moral, and emotional approaches to getting the audience to have the reaction that the author hopes for.

Rhetorical Appeals

Rhetorical appeals refer to ethos, pathos, and logos. These are classical Greek terms, dating back to Aristotle, who is traditionally seen as the father of rhetoric. To be rhetorically effective (and thus persuasive), an author must engage the audience in a variety of compelling ways, which involves carefully choosing how to craft his or her argument so that the outcome, audience agreement with the argument or point, is achieved. Aristotle defined these modes of engagement and gave them the terms that we still use today: logos, pathos, and ethos.

Logos: Appeal to Logic

Logic. Reason. Rationality. Logos is brainy and intellectual, cool, calm, collected, objective.

When an author relies on logos, it means that he or she is using logic, careful structure, and objective evidence to appeal to the audience. An author can appeal to an audience’s intellect by using information that can be fact checked (using multiple sources) and thorough explanations to support key points. Additionally, providing a solid and non-biased explanation of one’s argument is a great way for an author to invoke logos.

For example, if I were trying to convince my students to complete their homework, I might explain that I understand everyone is busy and they have other classes (non-biased), but the homework will help them get a better grade on their test (explanation). I could add to this explanation by providing statistics showing the number of students who failed and didn’t complete their homework versus the number of students who passed and did complete their homework (factual evidence).

Logical appeals rest on rational modes of thinking , such as

  • Comparison –  a comparison between one thing (with regard to your topic) and another, similar thing to help support your claim. It is important that the comparison is fair and valid – the things being compared must share significant traits of similarity.
  • Cause/effect thinking –  you argue that X has caused Y, or that X is likely to cause Y to help support your claim. Be careful with the latter – it can be difficult to predict that something “will” happen in the future.
  • Deductive reasoning –  starting with a broad, general claim/example and using it to support a more specific point or claim
  • Inductive reasoning –  using several specific examples or cases to make a broad generalization
  • Exemplification –  use of many examples or a variety of evidence to support a single point
  • Elaboration – moving beyond just including a fact, but explaining the significance or relevance of that fact
  • Coherent thought – maintaining a well organized line of reasoning; not repeating ideas or jumping around

Pathos: Appeal to Emotions

When an author relies on pathos, it means that he or she is trying to tap into the audience’s emotions to get them to agree with the author’s claim. An author using pathetic appeals wants the audience to feel something: anger, pride, joy, rage, or happiness.  For example, many of us have seen the ASPCA commercials that use photographs of injured puppies, or sad-looking kittens, and slow, depressing music to emotionally persuade their audience to donate money.

Pathos-based rhetorical strategies are any strategies that get the audience to “open up” to the topic, the argument, or to the author. Emotions can make us vulnerable, and an author can use this vulnerability to get the audience to believe that his or her argument is a compelling one.

Pathetic appeals might include

  • Expressive descriptions of people, places, or events that help the reader to feel or experience those events
  • Vivid imagery  of people, places or events that help the reader to feel like he or she is seeing  those events
  • Sharing  personal stories that make the reader feel a connection to, or empathy for, the person being described
  • Using emotion-laden   vocabulary  as a way to put the reader into that specific emotional mindset (what is the author trying to make the audience feel? and how is he or she doing that?)
  • Using any information that will evoke an emotional response from the audience . This could involve making the audience feel empathy or disgust for the person/group/event being discussed, or perhaps connection to or rejection of the person/group/event being discussed.

When reading a text, try to locate when the author is trying to convince the reader using emotions because, if used to excess, pathetic appeals can indicate a lack of substance or emotional manipulation of the audience. See the links below about fallacious pathos for more information.

Ethos: Appeal to Values/Trust

Ethical appeals have two facets: audience values and authorial credibility/character.

On the one hand, when an author makes an ethical appeal, he or she is attempting to  tap into the  values or ideologies that the audience holds , for example, patriotism, tradition, justice, equality, dignity for all humankind, self preservation, or other specific social, religious or philosophical values (Christian values, socialism, capitalism, feminism, etc.). These values can sometimes feel very close to emotions, but they are felt on a social level rather than only on a personal level. When an author evokes the values that the audience cares about as a way to justify or support his or her argument, we classify that as ethos. The audience will feel that the author is making an argument that is “right” (in the sense of moral “right”-ness, i.e., “My argument rests upon that values that matter to you. Therefore, you should accept my argument”). This first part of the definition of ethos, then, is focused on the audience’s values.

On the other hand, this sense of referencing what is “right” in an ethical appeal connects to the other sense of ethos: the  author. Ethos that is centered on the author revolves around two concepts: the credibility of the author and his or her character.

Credibility of the speaker/author is determined by his or her knowledge and expertise in the subject at hand. For example, if you are learning about Einstein’s Theory of Relativity, would you rather learn from a professor of physics or a cousin who took two science classes in high school thirty years ago? It is fair to say that, in general, the professor of physics would have more credibility to discuss the topic of physics. To establish his or her credibility, a n author may draw attention to who he or she is or what kinds of experience he or she has with the topic being discussed as an ethical appeal (i.e., “Because I have experience with this topic –  and I know my stuff! – you should trust what I am saying about this topic”). Some authors do not have to establish their credibility because the audience already knows who they are and that they are credible.

Character  is another aspect of ethos, and it   is different from credibility because it involves personal history and even personality traits. A person can be credible but lack character or vice versa. For example, in politics, sometimes the most experienced candidates – those who might be the most credible candidates – fail to win elections because voters do not accept their character. Politicians take pains to shape their character as leaders who have the interests of the voters at heart. The candidate who successfully proves to the voters (the audience) that he or she has the type of character that they can trust is more likely to win.

Thus, ethos comes down to trust. How can the author get the audience to trust him or her so that they will accept his or her argument? How can the the author make him or herself appear as a credible speaker who embodies the character traits that the audience values?

In building ethical appeals, we see authors

  • Referring either directly or indirectly to the values that matter to the intended audience (so that the audience will trust the speaker)
  • Using language, phrasing, imagery, or other writing styles common to people who hold those values, thereby “talking the talk” of people with those values (again, so that the audience is inclined to trust the speaker)
  • Referring to their experience and/or authority with the topic (and therefore demonstrating their credibility)
  • Referring to their own character, or making an effort to build their character in the text

When reading, you should always think about the author’s credibility regarding the subject as well as his or her character. Here is an example of a rhetorical move that connects with ethos: when reading an article about abortion, the author mentions that she has had an abortion. That is an example of an ethical move because the author is creating credibility via anecdotal evidence and first person narrative. In a rhetorical analysis project, it would be up to you, the analyzer, to point out this move and associate it with a rhetorical strategy.

 When writers misuse Logos, Pathos, or Ethos, arguments can be weakened

Above, we defined and described what logos, pathos, and ethos are and why authors may use those strategies. Sometimes, using a combination of logical, pathetic, and ethical appeals leads to a sound, balanced, and persuasive argument. It is important to understand, though, that using rhetorical appeals does not always lead to a sound, balanced argument.

In fact, any of the appeals could be misused or overused. When that happens, arguments can be weakened.

To see what a misuse of logical appeals might consist of, see the next chapter,   Logical Fallacies.

To see how authors can overuse emotional appeals and turn-off their target audience, visit the following link from WritingCommons.org :   Fallacious Pathos . 

To see how ethos can be misused or used in a manner that may be misleading, visit the following link to WritingCommons.org :  Fallacious Ethos

A Guide to Rhetoric, Genre, and Success in First-Year Writing by Melanie Gagich & Emilie Zickel is licensed under a Creative Commons Attribution-NonCommercial-ShareAlike 4.0 International License , except where otherwise noted.

Feedback/Errata

Comments are closed.

31 Useful Rhetorical Devices

What is a rhetorical device and why are they used.

As with all fields of serious and complicated human endeavor (that can be considered variously as an art, a science, a profession, or a hobby), there is a technical vocabulary associated with writing. Rhetoric is the name for the study of writing or speaking as a means of communication or persuasion, and though a writer doesn’t need to know the specific labels for certain writing techniques in order to use them effectively, it is sometimes helpful to have a handy taxonomy for the ways in which words and ideas are arranged. This can help to discuss and isolate ideas that might otherwise become abstract and confusing. As with the word rhetoric itself, many of these rhetorical devices come from Greek.

quill-in-ink

Ready, set, rhetoric.

The repetition of usually initial consonant sounds in two or more neighboring words or syllables

wild and woolly, threatening throngs

Syntactical inconsistency or incoherence within a sentence especially : a shift in an unfinished sentence from one syntactic construction to another

you really should have—well, what do you expect?

Repetition of a prominent and usually the last word in one phrase or clause at the beginning of the next

rely on his honor—honor such as his?

A literary technique that involves interruption of the chronological sequence of events by interjection of events or scenes of earlier occurrence : flashback

Repetition of a word or expression at the beginning of successive phrases, clauses, sentences, or verses especially for rhetorical or poetic effect

we cannot dedicate—we cannot consecrate—we cannot hallow—this ground

The repetition of a word within a phrase or sentence in which the second occurrence utilizes a different and sometimes contrary meaning from the first

we must all hang together or most assuredly we shall all hang separately

The usually ironic or humorous use of words in senses opposite to the generally accepted meanings

this giant of 3 feet 4 inches

The use of a proper name to designate a member of a class (such as a Solomon for a wise ruler) OR the use of an epithet or title in place of a proper name (such as the Bard for Shakespeare)

The raising of an issue by claiming not to mention it

we won't discuss his past crimes

An expression of real or pretended doubt or uncertainty especially for rhetorical effect

to be, or not to be: that is the question

Harshness in the sound of words or phrases

An inverted relationship between the syntactic elements of parallel phrases

working hard, or hardly working?

A disjunctive conclusion inferred from a single premise

gravitation may act without contact; therefore, either some force may act without contact or gravitation is not a force

The substitution of a disagreeable, offensive, or disparaging expression for an agreeable or inoffensive one

greasy spoon is a dysphemism for the word diner

Repetition of a word or expression at the end of successive phrases, clauses, sentences, or verses especially for rhetorical or poetic effect

of the people, by the people, for the people

Emphatic repetition [ this definition is taken from the 1934 edition of Webster's Unabridged dictionary ]

An interchange of two elements in a phrase or sentence from a more logical to a less logical relationship

you are lost to joy for joy is lost to you

A transposition or inversion of idiomatic word order

judge me by my size, do you?

Extravagant exaggeration

mile-high ice-cream cones

The putting or answering of an objection or argument against the speaker's contention [ this definition is taken from the 1934 edition of Webster's Unabridged dictionary ]

Understatement in which an affirmative is expressed by the negative of the contrary

not a bad singer

The presentation of a thing with underemphasis especially in order to achieve a greater effect : UNDERSTATEMENT

A figure of speech in which a word or phrase literally denoting one kind of object or idea is used in place of another to suggest a likeness or analogy between them ( Metaphor vs. Simile )

drowning in money

A figure of speech consisting of the use of the name of one thing for that of another of which it is an attribute or with which it is associated

crown as used in lands belonging to the crown

The naming of a thing or action by a vocal imitation of the sound associated with it

A combination of contradictory or incongruous words

cruel kindness

The use of more words than those necessary to denote mere sense : REDUNDANCY

I saw it with my own eyes

A figure of speech comparing two unlike things that is often introduced by "like" or "as"

cheeks like roses

The use of a word in the same grammatical relation to two adjacent words in the context with one literal and the other metaphorical in sense

she blew my nose and then she blew my mind

A figure of speech by which a part is put for the whole (such as fifty sail for fifty ships ), the whole for a part (such as society for high society ), the species for the genus (such as cutthroat for assassin ), the genus for the species (such as a creature for a man ), or the name of the material for the thing made (such as boards for stage )

The use of a word to modify or govern two or more words usually in such a manner that it applies to each in a different sense or makes sense with only one

opened the door and her heart to the homeless boy

MORE TO EXPLORE: Rhetorical Devices Used in Pop Songs

Word of the Day

See Definitions and Examples »

Get Word of the Day daily email!

Games & Quizzes

Play Quordle: Guess all four words in a limited number of tries.  Each of your guesses must be a real 5-letter word.

Usage Notes

Prepositions, ending a sentence with, hypercorrections: are you making these 6 common mistakes, a comprehensive guide to forming compounds, can ‘criteria’ ever be singular, singular nonbinary ‘they’: is it ‘they are’ or ‘they is’, grammar & usage, primary and caucus: what is the difference, words commonly mispronounced, merriam-webster’s great big list of words you love to hate, more commonly misspelled words, commonly misspelled words, 12 words for signs of spring, 12 more bird names that sound like insults (and sometimes are), 13 unusually long english words, the words of the week - apr. 26, 9 superb owl words.

The Rhetorical Question: a Powerful Tool in Communication

This essay about the power of rhetorical questions in communication explores their effectiveness in engaging audiences, stirring emotions, and guiding thoughts. It discusses how rhetorical questions prompt introspection, foster connection, and stimulate critical thinking. Additionally, the essay examines the role of rhetorical questions in storytelling, enhancing narratives with suspense, revelation, and nuance. Overall, it highlights the silent yet potent impact of rhetorical questions in shaping communication and forging connections between speakers and audiences.

How it works

Nestled within the vast landscape of communication techniques lies a gem that dazzles with its subtlety yet wields immense power: the rhetorical question. Unlike its straightforward counterpart, the rhetorical question is not seeking answers but rather inviting introspection, stirring emotions, and guiding minds towards a predetermined destination. From the grand stages of speeches to the intimate realms of everyday conversation, the strategic use of rhetorical questions infuses communication with depth, resonance, and persuasion, making it an indispensable tool in the arsenal of effective communicators.

At its essence, the rhetorical question is a silent conductor orchestrating a symphony of thought. It beckons the audience to step into the spotlight of introspection, nudging them towards a conclusion that aligns with the speaker’s intended message. Like a skilled navigator charting a course through uncharted waters, the speaker gently guides the audience’s thoughts and emotions, leading them on a journey of discovery that culminates in a shared understanding or conviction.

Yet, the true magic of the rhetorical question lies in its ability to evoke emotion and foster connection. By tapping into universal truths, values, or experiences, the speaker forges a bond with the audience, transcending the boundaries of words and logic. Whether stirring empathy, igniting passion, or fueling determination, rhetorical questions serve as conduits for the raw energy of human emotion, infusing communication with authenticity and resonance.

Moreover, rhetorical questions serve as catalysts for intellectual exploration and critical thinking. Like keys that unlock the gates of curiosity, they invite the audience to delve into the depths of their own minds, probing assumptions, challenging beliefs, and unraveling complexities. In the process, minds are sharpened, perspectives are broadened, and new insights emerge, enriching the discourse and deepening the audience’s engagement with the subject matter.

In the realm of storytelling, rhetorical questions wield a different kind of magic, transforming narratives into immersive experiences that captivate and enthrall. Whether dangling like tantalizing threads of suspense, punctuating moments of revelation, or echoing the whispers of inner conflict, rhetorical questions imbue stories with depth, texture, and nuance. They beckon the audience to lean in closer, to lean forward in anticipation, as they eagerly await the next twist or turn in the narrative.

In conclusion, the rhetorical question stands as a testament to the artistry of communication, weaving threads of thought, emotion, and connection into the fabric of human interaction. From the lofty heights of persuasion to the intimate whispers of storytelling, rhetorical questions leave an indelible mark on the landscape of communication, shaping opinions, inspiring action, and forging bonds that transcend words. As we navigate the complexities of communication, let us not overlook the silent power of the rhetorical question—a force to be reckoned with, a spark that ignites minds, and a bridge that connects hearts.

owl

Cite this page

The Rhetorical Question: a Powerful Tool in Communication. (2024, Apr 29). Retrieved from https://papersowl.com/examples/the-rhetorical-question-a-powerful-tool-in-communication/

"The Rhetorical Question: a Powerful Tool in Communication." PapersOwl.com , 29 Apr 2024, https://papersowl.com/examples/the-rhetorical-question-a-powerful-tool-in-communication/

PapersOwl.com. (2024). The Rhetorical Question: a Powerful Tool in Communication . [Online]. Available at: https://papersowl.com/examples/the-rhetorical-question-a-powerful-tool-in-communication/ [Accessed: 30 Apr. 2024]

"The Rhetorical Question: a Powerful Tool in Communication." PapersOwl.com, Apr 29, 2024. Accessed April 30, 2024. https://papersowl.com/examples/the-rhetorical-question-a-powerful-tool-in-communication/

"The Rhetorical Question: a Powerful Tool in Communication," PapersOwl.com , 29-Apr-2024. [Online]. Available: https://papersowl.com/examples/the-rhetorical-question-a-powerful-tool-in-communication/. [Accessed: 30-Apr-2024]

PapersOwl.com. (2024). The Rhetorical Question: a Powerful Tool in Communication . [Online]. Available at: https://papersowl.com/examples/the-rhetorical-question-a-powerful-tool-in-communication/ [Accessed: 30-Apr-2024]

Don't let plagiarism ruin your grade

Hire a writer to get a unique paper crafted to your needs.

owl

Our writers will help you fix any mistakes and get an A+!

Please check your inbox.

You can order an original essay written according to your instructions.

Trusted by over 1 million students worldwide

1. Tell Us Your Requirements

2. Pick your perfect writer

3. Get Your Paper and Pay

Hi! I'm Amy, your personal assistant!

Don't know where to start? Give me your paper requirements and I connect you to an academic expert.

short deadlines

100% Plagiarism-Free

Certified writers

Rhetorical analysis

Edit the Site Title and this tagline from the Dashboard > Appearance > Customize > Site Identity

FIQWS 10103

Rhetorical Analysis

Dear Professor,

During Phase 3 I learned how to break down stories using logos, pathos, and ethos. Before this I didn’t really know what they were. Now I understand all 3 of them. Logos for me are the hardest unless it’s directly in my face. Ethos is kind of hard but I believe it can be broken down. Pathos is the easiest and always was when I first heard of it. I didn’t realize how many of these things were in everyday life. For example, in class we saw an example of Ethos, Logos, and pathos on dog food. I get dog food frequently but I never really read the labels or information. Or even in videos where people do skits. They have a message behind them with ethos, logos, and pathos. Stuff that I do regularly have all of this and I was completely oblivious to it. Thanks to phase 3 I am able to point them out.

Even in readings I am able to point them out. Just like in my essay Amy Tan had a lot of pathos in her story. She also had ethos and credibility in her story. Even in other readings not included in this class. I am able to point these things out and put them into other papers I write. Phase 3 has really opened my eyes to see the emotion and experiences in people’s writings whether fictional or not. Phase 3 has also helped me make connections in readings to modern day struggles. Like people getting mistreated because they can’t speak Standard English or how big movie companies have hidden messages in them that some can be deemed offensive. I believe it is no easy task to put these ideas, emotions, and facts all together but looking at examples and connecting them to the readings in class I clearly see how the authors do it and how well they structure it. That is how Phase 3 has opened my eyes and opinions whether in real life or reading a passage.

Amy Tan’s Mother Tongue is a great example of language discrimination. The story is about the discrimination Amy Tan has experienced with her mother over time. There were two experiences where Amy Tan had to use Standard English to be heard and resolve an issue. In both examples Amy Tan’s mother was unable to speak or understand so Amy Tan had to take over. Amy Tan does a great job using pathos and ethos to show how people treat or talk to a person who doesn’t know Standard English.

The first example of Amy Tan using Standard English was when her mother had a problem with a stockbroker. She wanted Amy Tan to speak for her and pretend to be her because Amy Tan knew had to speak Standard English. When Amy Tan called she immediately was able to talk to the stockbroker clearly and was able to get a meeting with them. However, this time it was her mother that was talking in her broken English. The other example was when Amy Tan’s mother had gone to the doctor and they had lost her CAT scan. She refused to leave until the doctors called Amy Tan and sure enough once they did their attitude changed. They had apologized, made sure Amy Tan’s mother gets the quickest appointment, and assured the test would be found.

Amy Tan uses pathos to describe the first use of Standard English when dealing with the stockbroker. She uses her emotions of concern, nervousness, and even embarrassment at the end. She uses concern when talking to the stockbroker to make the situation a more serious one. She uses her nervousness because she knew she sounded nothing like her mother and her mother shouting in the background while Amy Tan was on the phone.She uses embarrassment when they go to New York and her mother is now speaking to the stockbroker in broken English. Amy Tan states “ And sure enough the following week there we were in front of this astonished stockbroker, and I was sitting there red-faced and quiet..” (pg.2). She was embarrassed that her mother and not her impersonating her mother was talking broken English to possibly the same stockbroker. However, if she had never done this, it would have been a different outcome.

Amy Tan uses pathos to connect the reader to her and her mothers experience. For example she uses emotions such as anger and annoyance but also confusion. She was annoyed that she had to use her Standard English to even get the right medical attention. She also must have been pretty angry that this was the way her mother was treated. She could have also been thinking what would have happened if she wasn’t able to speak in Standard English. Her mother probably would’ve been sent home without a diagnosis and maybe even something worse could have happened while she was waiting for that long appointment they gave her. Amy Tan did really well in getting the reader to be sympathetic and for me disgusted at the doctors who said they lost the CAT scan and there was nothing they could do. The reader could also be feeling the same way or had sadly gone through a similar experience. Amy Tan does a phenomenal job at getting the reader to not only feel a certain way about what they’re reading but also able to connect to the story with their own experience or something they experienced first hand.

 Amy Tan uses Ethos to make her mother’s experience credible and not something that was made up or something that could happen. She uses her mother’s experience by asking her what happened and telling the readers that story through her mother’s words. Another thing that makes her credible is that she is able to switch from 3rd person when she is sharing her mother’s story and 1st person when she is sharing her side of the story. Not only does this make the reader be able to understand the point she is trying to bring out but it also gives the reader the chance to experience their story and hardships through their very own eyes.

Amy Tan does well to connect to her audience in various ways. She was able to share her stories using emotion and detail while still holding the significance of her and her mother’s story. The way she is able to connect to the readers is special because her story is relatable to a lot of people during this day and age. Even though her type of writing may not be unique, the way she structures it and again shows the significance stands out from the rest of the stories we have read. The ability to connect language discrimination to things that are important like in this case medical care and assistance brings out the problem in our society now. In today’s world people are not receiving the care that they need either due to the fact of their race or how they speak. Some of these cases of neglect result in deaths that could have been prevented. This isn’t even the worst of it. The hospital and doctors are willing to cover up these cases in order to protect their title and the name of their institution. So for Amy Tan to speak up and put these neglectful actions out in public is really important for everyone who has been or who might be discriminated against in the future.

Attribution-NonCommercial-ShareAlike 4.0 International

This entry is licensed under a Creative Commons Attribution-NonCommercial-ShareAlike 4.0 International license.

Powered by WordPress / Academica WordPress Theme by WPZOOM

what is a rhetorical essay example

Need help with the Commons?

Email us at [email protected] so we can respond to your questions and requests. Please email from your CUNY email address if possible. Or visit our help site for more information:

CUNY Academic Commons logo

  • Terms of Service
  • Accessibility
  • Creative Commons (CC) license unless otherwise noted

CUNY logo

  • Share full article

A photo illustration of a bison smiling.

How Do We Know What Animals Are Really Feeling?

Animal-welfare science tries to get inside the minds of a huge range of species — in order to help improve their lives.

Credit... Photo Illustration by Zachary Scott

Supported by

By Bill Wasik and Monica Murphy

Bill Wasik is the magazine’s editorial director and Monica Murphy is a veterinarian and writer.

  • April 23, 2024

What makes a desert tortoise happy? Before you answer, we should be more specific: We’re talking about a Sonoran desert tortoise, one of a few species of drab, stocky tortoises native to North America’s most arid landscapes. Adapted to the rocky crevices that striate the hills from western Arizona to northern Mexico, this long-lived reptile impassively plods its range, browsing wildflowers, scrub grasses and cactus paddles during the hours when it’s not sheltering from the brutal heat or bitter cold. Sonoran desert tortoises evolved to thrive in an environment so different from what humans find comfortable that we can rarely hope to encounter one during our necessarily short forays — under brimmed hats and layers of sunblock, carrying liters of water and guided by GPS — into their native habitat.

Listen to this article, read by Gabra Zackman

This past November, in a large, carpeted banquet room on the University of Wisconsin’s River Falls campus, hundreds of undergraduate, graduate and veterinary students silently considered the lived experience of a Sonoran desert tortoise. Perhaps nine in 10 of the participants were women, reflecting the current demographics of students drawn to veterinary medicine and other animal-related fields. From 23 universities in the United States and Canada, and one in the Netherlands, they had traveled here to compete in an unusual test of empathy with a wide range of creatures: the Animal Welfare Assessment Contest.

That morning in the banquet room, the academics and experts who organize the contest (under the sponsorship of the American Veterinary Medical Association, the nation’s primary professional society for vets) laid out three different fictional scenarios, each one involving a binary choice: Which animals are better off? One scenario involved groups of laying hens in two different facilities, a family farm versus a more corporate affair. Another involved bison being raised for meat, some in a smaller, more managed operation and others ranging more widely with less hands-on human contact.

Then there were the tortoises. On screens along the room’s outer edge, a series of projected slides laid out two different settings: one, a desert museum exhibiting seven Sonoran specimens together in a large, naturalistically barren outdoor enclosure; the other, a suburban zoo housing a group of four tortoises, segregated by sex, in small indoor and outdoor pens furnished with a variety of tortoise toys and enticements. Into the slides had been packed an exhausting array of detail about the care provided for the tortoises in each facility. Only contestants who had prepared thoroughly for the competition — by researching the nutritional, environmental, social and medical needs of the species in question — would be able to determine which was doing a better job.

“Animal welfare” is sometimes misused as a synonym for “animal rights,” but in practice the two worldviews can sometimes be at cross purposes. From an animal rights perspective, nearly every human use of animals is morally suspect, but animal-welfare thinkers take it as a given that animals of all kinds do exist in human care, for better or worse, and focus on how to treat them as well as possible. In the past half century, an interdisciplinary group of academics, working across veterinary medicine and other animal-focused fields, have been trying to codify what we know about animal care in a body of research referred to as “animal-welfare science.”

The research has unlocked riddles about animal behavior, spurred changes in how livestock are treated and even brought about some advances in how we care for our pets: Studies of domestic cats, for example, have found that “puzzle feeders,” which slow consumption and increase mental and physical effort while eating, can improve their health and even make them friendlier. The discipline has begun to inform policy too, including requirements for scientists receiving federal grants for their animal-based research, regulations governing transport and slaughter of livestock, accreditation standards for zoos and aquariums and guidelines for veterinarians performing euthanasia.

Contest organizers hope to help their students, who might someday go into a range of animal-related jobs — not just as vets but in agribusiness, conservation, government and more — employ data and research to improve every aspect of animal well-being. Americans own an estimated 150 million dogs and cats, and our policies and consumption patterns determine how hundreds of millions of creatures from countless other species will live and die. The Animal Welfare Assessment Contest tries to introduce students to that enormous collective responsibility, and to the complexity of figuring out what each of these animals needs, especially when — as in the case of reptiles living in a shell — their outlook differs radically from our own.

The effort to improve the lives of America’s animals began more than 150 years ago. As it happens, a hundred or so turtles figured in one of the most important events in the early history of animal activism in America. It was May 1866 — the heyday of turtle soup, a dish so beloved at the time that restaurants in New York would take out newspaper ads, or even maintain special outdoor signage, declaring the hour at which the day’s batch would be ready. And so this group of unlucky sea turtles, after being captured by hunters in Florida, was brought to New York upside down on a schooner. To further immobilize them, holes were pierced through their fins with cords run through them.

The turtles would have assumed a tranquil, passive demeanor under such conditions, perhaps making it possible for the ship’s crew to believe that the creatures weren’t suffering. But there is every reason to believe they were. Evolution has equipped the marine turtle for a life afloat, with a large lung capacity filling the space beneath the shell, to enable long dives. When the turtles were on their backs, the weight of their organs would have put pressure on these lungs, forcing their breathing to become deliberate and deep.

The American Society for the Prevention of Cruelty to Animals started up the month before. Its president and founder, a Manhattan shipbuilding heir named Henry Bergh, spent its early weeks focusing on domestic species — above all, horses, the rough treatment of which in 19th-century streets was the main inspiration for his activism. But when he became aware of these suffering sea turtles for sale at Fulton Market, he decided to extend his campaign to a wildlife species that then barely rated more consideration than a cockroach, if not a cabbage.

Bergh made a case that the infliction of prolonged pain and distress upon sea turtles bound for the soup pot was illegal as well as immoral. As with other “mute servants of mankind” providing labor, locomotion, meat or milk to human beings, the turtle was entitled to be treated with compassion. But when Bergh hauled the ship’s captain in front of a judge, the defense argued (successfully!) that turtles were not even “animals,” but rather a form of fish, and thereby did not qualify under the new animal-cruelty law that Bergh succeeded in passing earlier that year.

A photo illustration of a rat smiling.

Still, the case became a media sensation — and signaled to New Yorkers that Bergh’s campaign on behalf of animals was going to force them to account for the suffering of all animals, not just the ones they already chose to care about.

It’s perhaps no accident that Bergh turned to activism after a failed career as a dramatist. There’s something irreducibly imaginative in considering questions of animal welfare, regardless of how much science we marshal to back up our conclusions. George Angell of Boston, his fellow titan of that founding generation of animal advocates, pirated a 13-year-old British novel called “Black Beauty” and turned it into one of the century’s best-selling books, touting it as “the ‘Uncle Tom’s Cabin’ of the Horse” — though its real innovation was its use of an animal as a first-person narrator, thrusting readers into a working horse’s perspective and forcing them to contemplate how the equines all around them might see the world differently.

But how far does imagination really get us? The philosopher Thomas Nagel famously explored this problem in an essay called “What Is It Like to Be a Bat?” which took up that question only to dramatize the impossibility of answering it to anyone’s satisfaction. “It will not help,” he wrote, “to try to imagine that one has webbing on one’s arms, which enables one to fly around at dusk and dawn catching insects in one’s mouth; that one has very poor vision, and perceives the surrounding world by a system of reflected high-frequency sound signals; and that one spends the day hanging upside down by one’s feet in an attic. Insofar as I can imagine this (which is not very far), it tells me only what it would be like for me to behave as a bat behaves.”

In the case of chelonians like turtles — and their encarapaced brethren, the tortoises — we may know even less about how they experience the world than we do about bats. Take their vision, for example: Among those species that have been studied, scientists have found evidence of broad-spectrum color vision, sometimes including ultraviolet wavelengths invisible to the human eye. And while chelonians can see well beyond their pointed beaks, where edible vegetation or predators may await notice, their brains process these visual signals slowly — a quality of certain animal brains that might, some experts have theorized, result in a sped-up perception of time. (In chelonian eyes, do grasses wave frenetically in the breeze and clouds race across the sky?)

Next to vision, smell is probably the sense turtles and tortoises rely upon most. Their sensitive nasal epithelium, distributed between two chambers, can detect odors diffused in a warm desert breeze or dissolved in a cold ocean current. Chelonian ears are where you’d expect them to be, but buried beneath their scaled reptilian skin. They hear well at low frequencies, even if they don’t register the high notes of twittering birds, humming mosquitoes or the whistling wind. Some chelonians seem to have the power of magnetoreception, which means that somewhere in their anatomy — perhaps their eyes, or their nervous systems, or elsewhere — there are chemicals or structures that allow them to sense the earth’s geomagnetic field and navigate by it.

The chelonian sense of touch presents fewer mysteries. Specialized receptors in the skin and on the shell detect mechanical, temperature and pain stimuli and send messages to the nervous system — just as they do in humans and a wide variety of other species. Recognition of pain, in particular, is considered a primordial sense, essential to the survival of animals on every limb of the evolutionary tree. But even here, there are differences separating species: What does the nervous system do with signals from its nociceptors? Does the desert tortoise withdraw its foot from the scorpion’s tail only reflexively, or does it consciously register the pain of the sting? What suffering does a turtle endure when its shell is struck by the sharp edges of a boat propeller?

As Nagel argued, there is no way to meaningfully narrow the gulch in understanding that exists around “what it is like to be” such a creature. The strategy of animal-welfare science is to patiently use what we can observe about these other kinds of minds — what they choose to eat and to do, how they interact with their environments, how they respond to certain forms of treatment — looking for objective cues to show experts what imagination cannot.

Upstairs from the banquet hall, student competitors nervously milled around carpeted corridors. One by one they were called into conference rooms to face a judge, who sat at a table beside a digital chronograph. In one room, a neatly dressed young woman in owlish glasses took a breath as the display began counting up hundredths of seconds in bright red digits. Catherine LeBlond, a second-year student at Atlantic Veterinary College at Canada’s University of Prince Edward Island, began her presentation about the bison scenario.

She was allowed to refer only to a single 3-by-5 index card, which she had packed with information based on a “summary sheet” of takeaways that she and her teammates worked up together, with key phrases emphasized and sources cited, all of it broken down by category: social behavior (“Bison are a very social species with strong matriarchal divisions”), handling guidelines (“Prods should not be used to move bison unless safety is an issue”), facility design (“Ensure that there is a sufficient number of gates within facilities to slow the animals”), euthanasia (“The recommended euthanasia method of a bison is gunshot”) and more.

LeBlond began by declaring her choice: The wilder facility provided a more humane environment for its animals. She felt it was helping bison “live a more natural life”: The more spacious grounds would support wallowing behavior, she reasoned, and allow the animals to choose their social grouping, an important policy given bisons’ strong sense of social structure. She also praised the operation for enabling bison to avoid “aversive life events,” by using drones, rather than ranchers on horseback, to monitor the animals in the field, and also by slaughtering the animals on-site to avoid the distress they experience in transport. As she ran through her presentation, she took care to hit two important rhetorical notes that judges look for: “granting” some areas in which the other institution excelled and offering positive advice about how it might improve.

One way to think about her reasoning is through the lens of “the five freedoms,” a rubric that animal-welfare thinkers have long embraced to consider all the different obligations that humans have to the animals in their care. They are: 1. the freedom from hunger and thirst; 2. the freedom from discomfort; 3. the freedom from pain, injury or disease; 4. the freedom to express normal behavior; and 5. the freedom from fear and distress. In fact, it was arguably the articulation of these five freedoms — in the Brambell Report, a document put out by a British government committee in 1965 to assess the welfare conditions of the nation’s livestock — that inaugurated the whole field of animal-welfare science.

What made this list of “freedoms” so influential, in retrospect, was that it created a context for other, more targeted thinking about how a species might experience each freedom or its violation. What sort of environment will offer “freedom from discomfort” to a beef steer, on the one hand, and a freedom “to express normal behavior” on the other? Trying to answer such questions in a rigorous way involves considerations of veterinary medicine but also of evolutionary history, behavioral observation, physiology (scientists have begun using proxies like cortisol levels as an indication of animal stress), neuroscience and more.

In her bison presentation, by citing “a more natural life” and avoiding “aversive life events,” LeBlond was emphasizing Freedoms 4 and 5, the freedom to express normal behavior and the freedom from distress. In the scenario about tortoises, though, Freedoms 4 and 5 seemed to be at odds. When LeBlond addressed the judge for that category, she awarded the edge to the zoo — weighing its better health outcomes and stimulating enrichments over the more naturalistic setting at the museum. She zeroed in on the zoo’s visitor program, which gave the tortoises a novel method of choosing whether or not they wanted to interact with humans: Staff put out a transport crate, and over the course of 20 minutes, tortoises could decide to climb into the crate to be taken to the human guests, and later receive a special biscuit for their service.

And she linked this to a behavioral difference, illustrated by a set of charts comparing how readily each set of tortoises moved toward a “novel object” (like an enrichment toy) or a “novel person” in their midst. The numbers showed that the zoo’s tortoises were far more drawn to interactions with people. “This indicates that they have less fear of humans,” LeBlond pointed out, “which could be because they are given a choice about whether or not they get to participate in educational programs, and those that do are positively reinforced with high-value rewards.”

Most of the students followed a similar logic and chose the zoo. The judges, however, disagreed. As one of them explained later at the awards ceremony — at which LeBlond took second place among vet students — the facility may have seemed to be offering their tortoises a consensual choice, but it was more accurate to see it as heavy-handed operant conditioning, which lured them into submitting to human contact with the promise of a biscuit. In scenarios involving domestic animals, a documented comfort around humans is a sign of positive treatment, but when it comes to wild animals, the goal is the opposite: to acclimate them as little to human contact as possible. Another way of putting it is this: Biscuits might make a desert tortoise “happy,” insofar as we can even imagine what that means, but happiness isn’t ultimately what humane treatment is about.

Each year at the contest, competitors are asked to perform one “live” assessment: a situation with real animals in it. This time, the species of choice was the laboratory rat. We joined Kurt Vogel, head of the Animal Welfare Lab at University of Wisconsin-River Falls, on a tour of the scenario that he and a colleague, Brian Greco, had constructed in a warren of rooms a few buildings over from the competition site.

They had brought a great deal of brio to the task. In the first room, where several rats snoozed in containers, Vogel and Greco had left a panoply of welfare infractions for eagle-eyed students to find. One cage was missing a water bottle, while others housed only a single rat, a violation of best practices (rats prefer to be housed in groups). Feed bags sat on the floor with detritus all around, and a note in a lab journal indicated that pest rodents had been observed snacking on it.

In subsequent rooms, the horrors became more baroque. A euthanasia chamber had the wrong size lid on it, and a nearby journal described a rat escaping in the middle of its extermination. Paperwork in an office laid out the nature of the study being performed, which involved prolonged deprivation of food and water, forced swimming and exposure to wet bedding. Diagrams showed that the rats’ brains were being studied through physical implants, and students could see that the operating room was a nightmare, littered with unsterile implements and the researchers’ food trash (the remnants of Vogel’s bagel sandwich, deliberately left behind). None of the abuse was real — Vogel and Greco were even taking care to cycle the rats in and out of the fake scenario, in order to avoid undue stress from the parade of students who came through taking notes.

Happiness isn’t ultimately what humane treatment is about.

Rodents did not always play the role in labs that they do today. In the late 19th century, experiments were carried out on a whole host of species, including a high proportion of dogs — a fact that animal-welfare activists publicized to turn the “vivisection” debate into a political issue, to the point that even some prominent doctors became galvanized to restrict or ban the practice. In the 20th century, as research shifted to carefully bred rats and mice, optimized for predictability and uniformity, animal experimentation receded as an issue in the public discourse. Today animal-welfare advocates struggle to motivate their base on the matter of rodents: the Humane Society’s website illustrates its section on “Taking Suffering Out of Science” (which sits at the very end in its list of the group’s current “fights”) with a picture of a beagle in a cage, despite the fact that roughly 95 percent of all lab mammals are now rats or mice.

Lab rodents are maybe the most vivid example of a species whose suffering is hard to know how to weigh against the benefits it provides us. Studies using rat and mouse models have sought to answer basic scientific questions across diverse fields of inquiry: psychology, physiology, pathology, genetics. Look into any new advance in human health care, and you’re likely to find that it’s built on years of experimentation that consumed the lives of literally thousands of rodents. We may now be on the cusp of innovations that could allow that toll to be radically reduced — by essentially replacing animal models with some combination of virtual simulations and lab-grown tissue and organs — but it’s hard to imagine a world anytime soon where human patients would be subject to therapies that have never been tested on hundreds of animals. No one even reliably counts how many rodents are killed in U.S. labs every year, but the estimates range from 10 million up to more than 100 million.

This question of scale especially haunts the problem of livestock, which is an area where many of the contest’s student competitors will eventually find jobs. America is currently home to roughly 87 million cattle and 75 million pigs: populations that exceed those of dogs and cats in scale, but the welfare of which commands so much less of our moral attention.

When the practice of centralized, industrialized livestock management began in earnest after the Civil War, the treatment of the animals, especially during slaughter, could be barbaric. Pigs were simply hoisted up and their throats cut, and after some point were assumed to be dead enough to dump into boiling water so that the sharp bristles on their skin could be scraped away. There was little doubt that some of them were still conscious at the point that they were plunged into the water, as was reported in a broad exposé in 1880 by The Chicago Tribune: “Not infrequently,” the reporter noted, “a hog reaches the scalding-tub before life is extinct; in fact, they sometimes are very full of life when they reach the point whence they are dumped into the seething tub.”

After 1906, when Upton Sinclair’s “The Jungle” exposed the industry’s unsanitary practices, a series of reforms did lead to significant improvements in the lives and deaths of American livestock. Thanks to the Humane Slaughter Act of 1958, federal law now requires that animals be “rendered insensible to pain” before the act of killing; with pigs, this is generally done either with electrocution or by suffocation in a carbon-dioxide chamber, while with cattle, the method of choice is the captive-bolt gun. And since the 1970s, animal-welfare science has led to some considerable reforms. Perhaps the most transformational work has been done by Temple Grandin, the animal behaviorist whose research into how food animals experience and respond to their environment — particularly during transport and slaughter — has changed the way that meat and dairy producers operate.

Still, despite years of promises to end the practice, many sows are still kept almost permanently in 7-feet-by-2-feet “gestation crates,” too small to turn around in. And the rise of concentrated animal feeding operations (CAFOs) has doomed millions of pigs, cattle and chickens to lives spent cheek to jowl in the stench of their own waste — waste that also threatens the health of nearby communities and ecosystems.

At the contest, many attendees were excited about the gains that artificial intelligence could bring to the animal-welfare field. Pilot studies have indeed shown great promise: For example, with A.I. assistance, 24-hour video surveillance can help pinpoint sick or injured animals much more quickly so they can be pulled out for veterinary care. Last year, a group of European researchers announced that based on 7,000 recordings of more than 400 pigs, they had made significant progress in understanding the meaning of their grunts. “By training an algorithm to recognize these sounds, we can classify 92 percent of the calls to the correct emotion,” one of the scientists remarked.

That well may be, but given what we know about pigs — specifically, their remarkable intelligence, which rivals (if not exceeds) that of a dog, to the point that a group of scientists recently trained some to play video games — there is no amount of A.I.-driven progress that can reconcile their short, crowded life as an American industrial food animal with any definition of what a “good” life looks like for such brainy creatures, all 75 million of them.

The laying hen, among the four species considered at the contest, is the one that lives among us in the largest numbers: There are an estimated 308 million of them in the United States alone, or nine for every 10 Americans. In a backyard flock, these hens could be expected to live six to eight years, but a vast majority of them toil in industrial operations that will slaughter them after only two to three years, once their productivity (six eggs a week) declines — and chickens, notably, are not covered by the Humane Slaughter Act. Poor air quality, soiled litter, nutritional stress and conflict with other chickens can contribute to dietary deficiencies, infectious diseases, egg-laying complications, self-mutilation, even cannibalism. And even in the best laying-hen operations, including the “cage-free” ones imagined in the contest scenario, these are short lives spent under 16 hours a day of artificial lighting in extremely close quarters with other birds.

More than in the other scenarios, the organizers had made the laying-hen choice a straightforward one. The corporate farm offered fewer amenities for the birds, which were also observed rarely to use the dirt-floored, plastic-covered “veranda” that was supposed to serve as a respite from their long hours laying in the aviary. The more commodious verandas of the family farm, covered with synthetic grass, proved more popular with their chickens, and in warm weather, its birds made use of a screened “garden” as well.

In her presentation, Catherine LeBlond correctly picked the family farm, for many of the same reasons that the judges did. Again, she “granted” some positive qualities of the corporate farm and offered it some advice — reflecting, after all, the values of the veterinary profession that she was training to enter, a field that takes on the advising of everyone who has animals in their care, not only the most conscientious.

Even so, at the very end, LeBlond briefly stepped back to ask a true ethical question, one that troubled the entire premise of a multibillion-dollar global industry: “whether or not it is ethical to keep these hens for the sole purpose of egg-laying, only to have them slaughtered at the end.” Among the scores of students we watched over the course of a weekend, LeBlond and her teammates from the Atlantic Veterinary College were the only ones who, in the final seconds of their talks, raised deep questions about the scenario’s entire premise — about whether, in the end, these fictional animals should have been put in these fictional situations in the first place.

It was a question that the judges of the Animal Welfare Assessment Contest had no doubt considered, but it also was one that seemed to lie outside the contest’s purview: In its either-or structure, the contest is helping train future professionals how to improve, rather than remove, the ties that bind animals into human society. Unless the day arrives when there is no need for laboratory rats, or poultry barns, or facilities to house desert tortoises and other captive wildlife, the animals of North America will be in the hands of veterinarians and animal scientists like LeBlond and her classmates, to help shape their situations the very best way they can.

Parts of this article are adapted from “Our Kindred Creatures: How Americans Came to Feel the Way They Do About Animals,” by Bill Wasik and Monica Murphy, published this month by Knopf.

Read by Gabra Zackman

Narration produced by Krish Seenivasan and Emma Kehlbeck

Engineered by Lance Neal

Explore The New York Times Magazine

Donald Trump’s Rally Rhetoric : No major American presidential candidate has talked like he now does at his rallies  — not Richard Nixon, not George Wallace, not even Trump himself.

The King of Tidy Eating : Rapturously messy food reviews are all over the internet. Keith Lee’s discreet eating style rises above them all .

What Are Animals Really Feeling? : Animal-welfare science tries to get inside the minds  of a huge range of species — in order to help improve their lives.

Can a Sexless Marriage Be Happy?: Experts and couples are challenging the conventional wisdom  that sex is essential to relationships.

Lessons From a 20-Person Polycule : Here’s how they set boundaries , navigate jealousy, wingman their spouses and foster community.

Advertisement

IMAGES

  1. How to Write a Great Rhetorical Analysis Essay: With Examples

    what is a rhetorical essay example

  2. BASIC OUTLINE of your rhetorical analysis essay

    what is a rhetorical essay example

  3. 60+ Rhetorical Devices with Examples for Effective Persuasion • 7ESL

    what is a rhetorical essay example

  4. How to Write a Rhetorical Analysis Introduction

    what is a rhetorical essay example

  5. Rhetorical Essay Outline : Rhetorical Analysis: the Basics

    what is a rhetorical essay example

  6. Rhetorical Analysis Essay Example

    what is a rhetorical essay example

VIDEO

  1. Rhetorical example (Needs Work)

  2. ENGL 101 Rhetorical Analysis PT 1

  3. Essay: Your Rhetorical Self

  4. The Rhetorical Analysis Essay

  5. How to Write a Rhetorical Analysis

  6. I Wrote an Ebook!

COMMENTS

  1. How to Write a Rhetorical Analysis

    A rhetorical analysis is a type of essay that looks at a text in terms of rhetoric. This means it is less concerned with what the author is saying than with how they say it: their goals, techniques, and appeals to the audience. A rhetorical analysis is structured similarly to other essays: an introduction presenting the thesis, a body analyzing ...

  2. How to Write a Rhetorical Analysis Essay-Examples & Template

    Rhetorical appeal #2: Pathos. The purpose of Pathos-driven rhetoric is to appeal to the reader's emotions. A common example of pathos as a rhetorical means is adverts by charities that try to make you donate money to a "good cause". To evoke the intended emotions in the reader, an author may use passionate language, tell personal stories ...

  3. How to Write a Great Rhetorical Analysis Essay: With Examples

    Name the author of the text and the title of their work followed by the date in parentheses. Use a verb to describe what the author does, e.g. "implies," "asserts," or "claims". Briefly summarize the text in your own words. Mention the persuasive techniques used by the rhetor and its effect.

  4. Rhetorical Analysis

    Rhetorical Analysis. Rhetoric is the study of how writers and speakers use words to influence an audience. A rhetorical analysis is an essay that breaks a work of non-fiction into parts and then explains how the parts work together to create a certain effect—whether to persuade, entertain or inform. You can also conduct a rhetorical analysis ...

  5. How to Write a Rhetorical Analysis Essay in 6 Steps

    How to Write a Rhetorical Analysis Essay in 6 Steps. Written by MasterClass. Last updated: Sep 2, 2021 • 3 min read. In a rhetorical analysis essay, a writer will examine the rhetoric and style of another author's work. If you want to write your own rhetorical analysis essay, we've developed a step-by-step guide to lead you through the ...

  6. How to Write a Rhetorical Analysis: 6 Steps and an Outline for Your

    Body Paragraph #1: Ethos. Describe how the speaker makes an appeal to ethos (the audience's sense of ethical responsibility) Use specific examples, referring to word choice, tone, anecdotes, and other devices. Body Paragraph #2: Pathos. Describe how the speaker makes an appeal to pathos (the audience's emotions)

  7. How to write a rhetorical analysis [4 steps]

    To write a rhetorical analysis, you need to follow the steps below: Step 1: Plan and prepare. With a rhetorical analysis, you don't choose concepts in advance and apply them to a specific text or piece of content. Rather, you'll have to analyze the text to identify the separate components and plan and prepare your analysis accordingly.

  8. How to Write a Rhetorical Analysis Essay

    2. Introducing Your Essay Topic. Introduce your essay by providing some context about the text you're analyzing. Give a brief overview of the author, intended audience, and purpose of the writing. You should also clearly state your thesis, which is your main point or argument about how and why the author uses rhetorical strategies.

  9. PDF How to Write a RHETORICAL ANALYSIS ESSAY Step 1: Full Comprehension of

    Like all other essays, your rhetorical analysis essay will have an introduction with a thesis, body paragraphs, and a conclusion. WRITE - write your essay. Asher AP ELAC Name: _____ Step 3: Organizing and Writing Your Essay: Some of this is redundant, but this breaks down some of the steps from MAD TO WRITE even further. ...

  10. How to Write a Rhetorical Analysis

    A rhetorical analysis essay is a type of essay where you examine how authors or speakers use words, phrases, and other techniques to influence or persuade their audience. This type of essay focuses on analyzing the strategies used by the writer or speaker to achieve their purpose, whether it's to inform, persuade, entertain, or provoke.

  11. How to Write the AP Lang Rhetorical Analysis Essay (With Example)

    The AP Lang Rhetorical Analysis Essay is one of three essays included in the written portion of the AP English Exam. The full AP English Exam is 3 hours and 15 minutes long, with the first 60 minutes dedicated to multiple-choice questions. Once you complete the multiple-choice section, you move on to three equally weighted essays that ask you ...

  12. 6.3 What is Rhetorical Analysis?

    One of the elements of doing a rhetorical analysis is looking at a text's rhetorical situation. The rhetorical situation is the context out of a which a text is created. The questions that you can use to examine a text's rhetorical situation are in Chapter 6.2. Another element of rhetorical analysis is simply reading and summarizing the text.

  13. Sample Rhetorical Analysis

    Read the following sample rhetorical analysis of an article. If you like, you can read the original article the student analyzes: Why I won't buy an iPad (and think you shouldn't, either). Then, click the image below to open a PDF of the sample paper. The strategies and techniques the author used in this rhetorical analysis essay have been ...

  14. Rhetorical Analysis Sample Essay

    Rhetorical Analysis Sample Essay. A woman's work is never done: many American women grow up with this saying and feel it to be true. 1 One such woman, author Jessica Grose, wrote "Cleaning: The Final Feminist Frontier," published in 2013 in the New Republic, 2 and she argues that while the men recently started taking on more of the ...

  15. Rhetorical Analysis Definition and Examples

    Rhetorical analysis is a form of criticism or close reading that employs the principles of rhetoric to examine the interactions between a text, an author, and an audience. It's also called rhetorical criticism or pragmatic criticism. Rhetorical analysis may be applied to virtually any text or image—a speech, an essay, an advertisement, a poem ...

  16. What Is Rhetoric? Definition, Examples, and Importance

    Definition, Examples, and Importance. Politicians deliver rallying cries to inspire people to act. Advertisers create catchy slogans to get people to buy products. Lawyers present emotional arguments to sway a jury. These are all examples of rhetoric—language designed to motivate, persuade, or inform.

  17. Sample Rhetorical Analysis Essay

    Sample Rhetorical Analysis Essay. Student essay is used with permission. It was originally submitted double-spaced with no extra spaces between the lines, featured proper MLA pagination, and 1/2″ paragraph indents. The writing assignment asks for an argument about how several rhetorical elements work together to create a functioning whole in ...

  18. 9.5 Writing Process: Thinking Critically about Rhetoric

    Discuss the strategies used, provide examples and quotations as appropriate, and show how they support (or don't support) the writer's thesis statement. Consider rhetorical strategies such as parallelism, repetition, rhetorical questions, and figurative language. Body paragraphs 4-6 (or more if needed) Continue as needed.

  19. 20+ Best Rhetorical Analysis Essay Example & Sample Papers

    Good Rhetorical Analysis Essay Example. The step-by-step writing process of a rhetorical analysis essay is far more complicated than ordinary academic essays. This essay type critically analyzes the rhetorical means used to persuade the audience and their efficiency. The example provided below is the best rhetorical analysis essay example:

  20. 17 Rhetorical Modes for Paragraphs & Essays

    For example, under the umbrella of an argument essay, and author might choose to write paragraphs showing cause and effect, description, and narrative. The rhetorical mode writers choose depends on the purpose for writing. Rhetorical modes are a set of tools that will give you greater flexibility and effectiveness in communicating with your ...

  21. 6.4 Rhetorical Appeals: Logos, Pathos, and Ethos Defined

    4.10 A review of the five-paragraph essay; 4.11 Moving Beyond the five-paragraph format; Deeper Reading: "I Need You to Say I" Chapter 5: Writing a Summary and Synthesizing. ... Here is an example of a rhetorical move that connects with ethos: when reading an article about abortion, the author mentions that she has had an abortion. ...

  22. 31 Common Rhetorical Devices and Examples

    An expression of real or pretended doubt or uncertainty especially for rhetorical effect. to be, or not to be: that is the question. cacophony | see definition ». Harshness in the sound of words or phrases. chiasmus | see definition ». An inverted relationship between the syntactic elements of parallel phrases.

  23. The Rhetorical Question: a Powerful Tool in Communication

    Essay Example: Nestled within the vast landscape of communication techniques lies a gem that dazzles with its subtlety yet wields immense power: the rhetorical question. Unlike its straightforward counterpart, the rhetorical question is not seeking answers but rather inviting introspection,

  24. John F Kennedy Rhetorical Analysis Essay

    John F Kennedy Rhetorical Analysis Essay. 451 Words 2 Pages. ... Logos, meaning an appeal to the audience's logic, can be found in several examples in this piece. President Kennedy uses the lack of the steel companies' wisdom to point out all the elements affected by increasing steel prices. He describes how the price of steel affects ...

  25. Rhetorical analysis

    Rhetorical Analysis. 10/25/2022. Dear Professor, ... For example, in class we saw an example of Ethos, Logos, and pathos on dog food. I get dog food frequently but I never really read the labels or information. Or even in videos where people do skits. ... Just like in my essay Amy Tan had a lot of pathos in her story. She also had ethos and ...

  26. How Do We Know What Animals Are Really Feeling?

    The philosopher Thomas Nagel famously explored this problem in an essay called "What Is It Like to ... she took care to hit two important rhetorical notes that judges look for: "granting ...